Unser Universum dehnt sich aus. Alles bewegt sich von allem weg und zwar um so schneller, je weiter es weg ist. Das wissen wir seit den revolutionären Beobachtungen von Edwin Hubble und diese Beobachtungen bilden die Grundlage der UrknallTheorie die die Entwicklung unseres Universums beschreibt. Die Geschwindigkeit mit der sich das Universum ausdehnt, wird durch die sogenannte Hubble-Konstante beschrieben. Die genau zu messen ist knifflig – aber sehr wichtig. Denn eine genaue Kenntnis dieser Zahl hilft nicht nur beim Verständnis der Expansion des Universums; man kann daraus auch Rückschlüsse auf die Gültigkeit einiger kosmologischer Hypothesen ziehen. Astronomen ist es nun kürzlich gelungen die Hubble-Konstante so genau wie nie zuvor zu messen und damit eine Alternative zur Theorie der dunklen Energie zu widerlegen.

Die Hubble-Konstante ist eine Zahl, die in Kilometer pro Sekunde pro Megaparsec angegeben wird (ein Parsec ist eine Entfernungseinheit und entspricht dabei etwa 3.26 Lichtjahren). Die Messung der Konstante (die mit H0) bezeichnet wird ist schwierig und anfangs hatte man dabei große Schwierigkeiten.

i-d941d91e08efaf59e986c543c4347f5a-hubble-plot-thumb-500x303.jpg
Hubbles berühmte Messungen die die Expansion des Alls belegen (Hubble et al. 1929)

Um überhaupt eine Chance zu haben, die Entfernung einer anderen Galaxie zu bestimmen, ist man darauf angewiesen, die Helligkeit bestimmter Sterne dort zu beobachten. Das allein reicht aber noch nicht. Denn wenn die Sterne einer Galaxie uns vergleichsweise hell erscheinen, dann kann das entweder daran liegen, dass die Galaxie uns sehr nahe ist – oder aber sie ist weit weg und die Sterne sind einfach sehr groß und deswegen heller als wir dachten. Ohne Details über den physikalischen Hintergrund und Kenntnis der absoluten Helligkeit kann man über die Entfernung nichts aussagen. Kennt man aber die absolute und die scheinbare Helligkeit, dann kann man daraus leicht die Entfernung berechnen. Ein Fall bei dem das funktioniert, sind die Cepheiden. Das ist eine Klasse von Sternen deren Helligkeit sich periodisch ändert und zwar nach einem ganz speziellen Muster das von ihrer absoluten Helligkeit abhängt. Kennt man also ihre Perioden und scheinbaren Helligkeiten, kann man daraus die absolute Helligkeit und damit ihre Entfernung berechnen. Genauso hat damals auch Edwin Hubble die Entfernung zur Andromeda-Galaxie bestimmt.

Ein zweiter Schritt auf der sogenannten “Entfernungsleiter” der Astronomie sind die Supernovae vom Typ Ia. Sternexplosionen von diesem Typ verlaufen ebenfalls immer nach einem ganz speziellen Muster aus dem sich ihre absolute Helligkeit ableiten lässt. Und da sie so extrem hell sind, kann man sie auch noch in weit entfernten Galaxien sehen. Man fängt also damit an, die Entfernung von Cepheiden in nahen Galaxien zu bestimmen. Idealerweise sind das Galaxien, in den man auch Supernovae vom Typ Ia beobachtet hat. Dann kann man die Cepheiden benutzen um die Entfernungsbestimmung mit den Supernovae zu kalibrieren und dann in den weiter entfernten Galaxien, in denen man keine Cepheiden mehr sehen kann, nach neuen Supernovae suchen. Jetzt muss man noch bestimmen, wie schnell sich die jeweiligen Galaxien von uns weg bewegen. Auch das geht durch die Beobachtung von Cepheiden bzw. Supernovae. Hier benutzt man die Spektroskopie und spaltet das Licht in seine Bestandteile auf. Dann sucht man nach den dunklen Spektrallinien (die durch das Vorhandensein der verschiedenen Elemente) und sieht nach, ob sie dort sind, wo sie sein sollen. Das ist normalerweise nicht der Fall – denn so wie der Dopplereffekt dafür sorgt dass sich die Tonhöhe bei bewegten Schallquellen (z.B. einem vorbeifahrenden Krankenwagen) ändert, verschieben sich auch die Spektrallinien im Licht das von sich bewegenden Galaxien ausgesandt wird.

Man kann nun also durch die Beobachtung von Cepheiden und Supernovae sowohl die Entfernung als auch die Geschwindigkeit vieler Galaxien bestimmen und wenn man dann noch ordentlich rechnet, dann erhält man einen Wert für die Hubble-Konstante. Diese Beobachtungen und Rechnungen haben kürzlich Wissenschaftler um Adam Riess (ich spar mir mal den Witz 😉 ) von der John Hopkins Universität in Baltimore gemacht. Die Ergebnisse sind in einem Artikel mit dem Titel A 3% solution: Determination of the Hubble Constant with the Hubble Space Telescope and Wide Field Camera veröffentlicht wurden. Die Beobachtungen dort waren Teil des “Supernovae and H0 for the Equation of State”-Projekts das auch unter dem wunderbaren Akronym SH0ES bekannt ist. Mit der Wide-Field-Kamera des Hubble-Weltraumteleskops wurden über 600 Cepheiden beobachtet die sich in acht Galaxien befanden in denen man schon Typ-Ia Supernovae kannte. Dann hat man nochmal 253 weitere Supernovae in entfernteren Galaxien beobachtet.

i-05ea29e71b18ccab0e8b07277e1d39fc-cephgalax-thumb-500x477.jpg
Cepheiden in der Galaxie NGC 5584 – verschiedene Farben zeigen verschiedene Perioden an (Bild: Riess et al. 2011).

Am Ende hatte man einen Wert für die Hubble-Konstante von 73.8 ± 2.4 Kilometer pro Sekunde pro Megaparsec bestimmt. Eine Galaxie in einer Entfernung von einem Megaparsec bewegt sich von uns also mit 73.8 Kilometern pro Sekunde (immerhin 265700 km/h) weg. Eine Galaxie in einer Entfernung von 2 Megaparsec ist doppelt so schnell: 147.6 Kilometer pro Sekunde – usw. Diese Meßergebnisse passen gut zu den theoretischen Vorhersagen die man durch die Beobachtung der kosmischen Hintergrundstrahlung gewonnen hatte. Die sagte einen Wert von 71.0±2.5 Kilometern pro Sekunde pro Megaparsec voraus und innerhalb der Fehlergrenzen stimmen die Werte überein. Die neuen Ergebnisse passen auch gut zu den früheren Messungen: bei den letzten genauen Beobachtungen aus dem Jahr 2009 hat man 74.2 +/- 3.6 Kilometer pro Sekunde pro Megaparsec gemessen. Auch das stimmt innerhalb der Fehlergrenzen mit dem neuen Wert überein nur das bei dem die Fehlergrenzen kleiner sind.

Der größte Beitrag zu diesem Fehler wird übrigens von der geringern Zahl an Kalibrationssupernovae (das waren ja nur acht) verursacht. Man hofft aber, dass zukünftige Durchmusterungen mehr passende Supernovae finden werden und das neue Weltraumteleskope wie GAIA oder JWST demnächst mehr Cepheiden auch weiter weg beobachten können.

In der Arbeit der Astronomen findet sich am Schluss auch noch folgender interessanter Satz:

“The measured H0 is also highly inconsistent with the simplest inhomogeneous matter models invoked to explain the apparent acceleratio nof the universe without dark energy.”

Der nun gemessene Wert der Hubble-Konstante ist also höchst inkonsistent mit einer der Alternativtheorie zur dunklen Energie. Denn als 1998 Wissenschaftler schonmal Typ-Ia Supernovae beobachteten um Entferung und Geschwindigkeit zu bestimmen, fanden sie etwas seltsames heraus: die Expansionsgeschwindigkeit des Universums ist nicht konstant, sondern sie nimmt zu! Den Grund dafür kennt man noch nicht – auch wenn es jede Menge Hypothesen gibt. Die Existenz dieser dunklen Energie ist durch unabhängige Beobachtungen relativ zwingend belegt – aber trotzdem gab es bis jetzt auch noch eine alternative Deutung der Messungen. Die besagte, dass sich unsere Milchstrasse zufällig in einer sehr großen, etwa acht Milliarden Lichtjahre durchmessenden relativ leeren “Blase” befindet. Wenn wir dann noch in der Nähe der Mitte dieser Blase leben würden, dann könnte das den Eindruck der beschleunigten Expansion bewirken. Aber diese Hypothese konnte nun mit den neuen Daten von Riess et al. widerlegt werden. Was immer auch sich am Ende als Erklärung für die dunkle Energie herausstellen wird: ein einfacher Beobachtungseffekt ist es nicht!


Flattr this

Kommentare (186)

  1. #1 Björn Reinhardt
    29. März 2011

    Wie schnell bewegt sich dann eine Galaxie von uns weg, die 4100 Megaparsec entfernt ist?

  2. #2 Florian Freistetter
    29. März 2011
  3. #3 Gerry
    29. März 2011

    Einer der Artikel für die ich dieses Blog so liebe!

    Ein interessantes Thema das kurz und allgemeinverständlich aufbereitet wird und gleichzeitig noch Einblick in aktuelle Forschung bietet.

    Danke!

    Ich glaube ich muss doch noch nen Flattr-Account einrichten…

    Florian, gehen die Flattr-Einnahmen direkt an dich?

  4. #4 Björn Reinhardt
    29. März 2011

    Thx a lot! Aber schwierig zu verstehen.

    Hätte dann auch gedacht, dass das olberssche Paradoxon (und die einhergehende Begründung für ein endliches Universum) damit doch aufgelöst werden könnte, wie ich aber gerade eben gelernt habe, gilt das Argument schon lange nur noch als historisch.

    Erstaunlich, wie viel Halbwissen man mit sich herumträgt…

  5. #5 zwutz
    29. März 2011

    Na immerhin war die Blasentheorie ein netter Versuch zur Erklärung.

    Aber da das ja jetzt widerlegt ist, kann die Suche nach der dunklen Energie ja weitergehen.

  6. #6 Manea-K
    29. März 2011

    Achtung, Verwechslungsgefahr: Im letzten Absatz hast Du einmal dunkle Materie statt dunkle Energie geschrieben.

  7. #7 waaahsabi
    29. März 2011

    Frage: Ich stosse des Oefteren auf die Formulierung “nach dem Urknall”. Wenn ich jedoch lese, dass das Universum sich offenbar immer schneller ausdehnt, warum ist der Urknall dann ueberhaupt vorbei?

    Nur so ein Gedanke, denn wenn ich zb. eine x-beliebige Explosion betrachte, gibt es da ja auch erstmal eine Beschleunigungsphase, innerhalb derer sich das explodierende Objekt immer schneller ausdehnt. Ich koennte mir vorstellen, dass wir noch mittendrin sind in dieser Expansionsphase. Evtl dauert das halt nur etwas laenger 😉

    Oder hab ich da was misinterpretiert?

  8. #8 Lukas
    29. März 2011

    Ich muss wirklich sagen. Floran schafft es immer wieder mir als Laien Fakten über Astronomie so gut zu erklären, das ich sie verstehe. Bin echt schwer beeindruck, da das ganze ja eine sehr verwissenschaftliche Materie ist.
    Astronomie ist und bleibt auch dank dir Florian eines meiner größten Steckenpferde 🙂

    mach weiter so mit deiner tollen Arbeit!!!

    lg

  9. #9 Florian Freistetter
    29. März 2011

    @Gerry: “Florian, gehen die Flattr-Einnahmen direkt an dich? “

    Yep 😉

  10. #10 Bullet
    29. März 2011

    @waaahsabi:
    nicht ganz. Die Expansion verlief den Modellen zufolge am Anfang deutlich ruckartig und kam dann zum fast-Erliegen.
    Siehe dieses Bild:
    https://de.wikipedia.org/w/index.php?title=Datei:Expansion_des_Universums.png

  11. #11 Florian Freistetter
    29. März 2011

    @waaahsabi: Naja – “Urknall” ist sowieso ein blödes Wort 😉 Aber im Allgemeinen bezechnet man damit nicht die Expansion des Universums sondern die erste, ganz extreme Phase des Universms: https://www.scienceblogs.de/astrodicticum-simplex/2010/05/was-knallt-beim-urknall-inflation-und-expansion-im-universum.php

  12. #12 MartinB
    29. März 2011

    Wider legt das auch das Wiltshire-Modell (darüber waren wir uns neulich nicht so klar)?
    https://www.scienceblogs.de/hier-wohnen-drachen/2011/03/gibt-es-die-dunkle-energie.php

  13. #13 Bjoern
    29. März 2011

    @Florian:

    Naja – “Urknall” ist sowieso ein blödes Wort 😉 Aber im Allgemeinen bezechnet man damit nicht die Expansion des Universums sondern die erste, ganz extreme Phase des Universms

    Das ganze wird dadurch noch komplizierter, dass die Urknall-Theorie trotz ihres Namens eigentlich nicht den Urknall selber (also den Anfang des Universums) beschreibt, sondern wie es sich danach entwickelt hat… (von den ersten Sekundenbruchteilen bis heute).

  14. #14 Oliver Debus
    29. März 2011

    Toller Artikel.

    der Begriff Urknall-Theorie sollte doch eher Theorie von der Entstehung und Entkicklung der Universums heissen. Leider hat sich der mißverständliche Titel, den sich die Gegner der Theorie ausgedacht haben durchgesetzt, wenn ich mich nicht irre, hihi.

  15. #15 Reinhard
    29. März 2011

    Ich hätt da eine Frage: Dehnt sich das Univerum eigentlich geradlinig aus?

  16. #16 Hajo Becker
    29. März 2011

    Die Genauigkeit von +/- 2,4 km/s/Mpc ist schon beeindruckend. Was mich allerdings dabei etwas irritiert: für welche Entfernungen (oder Rotverschiebungen oder was auch immer) gilt der gemessene Wert von 73.8 ? Schließlich ist die Hubble-“Konstante” ja offenbar nicht konstant – sonst bräuchte man ja auch die dunkle Energie nicht. Beim Überfliegen des Originals in ApJ habe ich dazu auch nichts wirklich Brauchbares gefunden.

  17. #17 Florian Freistetter
    29. März 2011

    @MartinB: Da bin ich momentan erstmal überfragt… Wiltshire gibts ja schon länger; wahrscheinlich hätten die Autoren das erwähnt, wenn das widerlegt worden wäre.

  18. #18 Florian Freistetter
    29. März 2011

    @Hajo Becker: Naja – mit Entfernung ists immer kritisch in der Astronomie weil das ja gleichzeitig auch immer in der Vergangenheit liegt. Ich gehe mal davon aus, dass die 73.8 der aktuelle Wert sind. Sie ist ja auch als Expansionsrate des aktuellen Universums definiert.

  19. #19 Bjoern
    29. März 2011

    @Hajo Becker:

    für welche Entfernungen (oder Rotverschiebungen oder was auch immer) gilt der gemessene Wert von 73.8 ?

    Für alle. Egal welche Entfernung / Rotverschiebung: H mal (heutige!) Entfernung ergibt die (heutige!) Geschwindigkeit des Objekts von uns weg (wobei der Begriff “Geschwindigkeit” hier auch mit Vorsicht zu genießen ist – eigentlich bewegen sich die Objekte ja nicht, sondern sie entfernen sich nur von uns, weil sich der Raum zwischendrin ausdehnt).

    Schließlich ist die Hubble-“Konstante” ja offenbar nicht konstant –

    In der Tat ist sie das nicht – das ändert aber nix an dem oben gesagten. (ich weiß, ist nicht so leicht zu verstehen… der springende Punkt ist das zweimalige “heutige” in der Formulierung oben)

    – sonst bräuchte man ja auch die dunkle Energie nicht.

    Das hat wenig mit der dunklen Energie zu tun – auch ohne dunkle Energie wäre die Hubble-Konstante nicht (zeitlich) konstant.

  20. #20 Bjoern
    29. März 2011

    @Hajo Becker: Was mir hilft, ist, statt der üblichen Einheit “km pro s und Mpc” die Hubble-Konstante in andere Einheiten umzurechnen. Dann bekommt man nämlich heraus, dass mir H letztlich sagt, um wie viel Prozent sich das Universum pro Zeiteinheit ausdehnt. Für den erwähnten Zahlenwert von 73,8 km/s/Mpc erhält man übrigens einen Wert von etwa 7 Prozent pro Milliarde Jahre. (natürlich ist auch dieser Prozentwert nicht konstant – aber im Laufe einer Milliarde Jahre ändert er sich kaum, also ist das eine gute Näherung)

  21. #21 Bjoern
    29. März 2011

    @Reinhard: Tut mir leid, deine Frage ist unklar. Was meinst du?

  22. #22 Saasaa
    29. März 2011

    @FF

    Siehe hier: https://en.wikipedia.org/wiki/Faster-than-light#Universal_expansion

    Was geschieht eigentlich wenn eine Galaxie hinter den Ereignishorizont fällt, wie es in dem Abschnitt erklärt worden ist? Verschwindet sie dann einfach vom Himmel?

  23. #23 Super Tanooki
    29. März 2011

    Ich habe nicht wirklich viel Ahnung von dem was hier steht und verstehe dementsprechend auch recht wenig. Aber ich habe trotzdem mal ne Frage:

    Also das Universum dehnt sich ja aus, richtig? Wird es sich denn ewig ausdehnen? Oder wird es irgendwann wie ein Gummiband, wenn man es zu weit dehnt “reißen”?

    Oder was könnte dabei alles irgendwann passieren, wenn es sich ständig ausdehnt?

  24. #24 Florian Freistetter
    29. März 2011

    @Saasaa: Sie würde zuerst immer röter werden und wäre dann nicht mehr sichtbar.

    @Super Tanooki: “Also das Universum dehnt sich ja aus, richtig? Wird es sich denn ewig ausdehnen? “

    Gute Frage! Das fragen sich die Wissenschaftler auch und eine definitive Antwort gibt es noch nicht. Es hängt alles von der Gesamtmasse im Universum ab. Ist die groß genug, dann wird die Gravitationskraft irgendwann dafür sorgen, dass sich das Universum wieder zusammenzieht. Wenn nicht, dann dehnt es sich immer weiter aus. “Reissen” kann da nicht wirklich was. Aber irgendwann, in ein paar Billiarden Billiarden Jahren wird es vielleicht einen “Big Rip” geben wenn nämlich die Expansionskraft stärker als die lokalen Gravitationskräfte werden die z.B. die Erde zusammenhalten (die es dann aber eh nicht mehr gibt) bzw. die Kernkräfte, die die Atome zusammenhalten.

  25. #25 MartinB
    29. März 2011

    @SuperTanooki
    Was sich da dehnt, ist ja der Raum. Vielleicht hilft dir meine kleine Raumzeit-Serie weiter:
    https://www.scienceblogs.de/hier-wohnen-drachen/2011/01/raumzeitkrummung-ganz-einfach.php

  26. #26 Bjoern
    29. März 2011

    @Super Tanooki: Das Universum (der “Raum”) selbst kann nicht reißen – es gibt aber sehr wohl Hypothesen darüber, dass es die Objekte im Raum irgendwann einmal zerreißen könnte:
    https://de.wikipedia.org/wiki/Big_Rip
    (aber bitte beachte: das ist hochspekulativ!)

  27. #27 Super Tanooki
    29. März 2011

    @ Florian Freistetter und MartinB

    Das hört sich echt interessant an. Ich glaube ich sollte mich auch mal damit beschäftigen. Aber ich müsste eher klein anfangen : ) Irgendwas, was auch jedes Kind verstehen würde. Ich finds einfach nur abgefahren was im Universum / Weltraum alles abgeht und welche Kräfte usw da im Spiel sind. Wenn ich versuche mir das irgendwie alles vorzustellen, gibts ganz schön Gedankenchaos bei mir im Kopf. Ich müsste es irgendwie alles Schritt für Schritt verstehen. Könnt ihr mir da vielleicht etwas empfehlen um in die Richtung einzusteigen? Oder ist das als 22 Jähriger, Hartz 4 Empfänger mit Hauptschulabschluss unmöglich, das von zu Hause aus mit Büchern, Dokus oder Seiten wie hier, zu verstehen? : )

  28. #28 Bjoern
    29. März 2011

    @Super Tanooki: Du könntest es mal mit “Kosmologie für die Westentasche” von Rudolf Kippenhahn versuchen. Ein schönes kleines Buch, in dem alles Wesentliche erklärt wird (und zwar ohne jegliche Formeln).

  29. #29 Bjoern
    29. März 2011

    @Florian: Nitpick zum Titel deines Artikels: Die Hubble-Konstante ist nicht die Expansions”geschwindigkeit” des Universums – mehr so etwas wie die Expansionsrate

  30. #30 Florian Freistetter
    29. März 2011

    @Super Tanooki: Du kannst auch mal “Der Stoff aus dem der Kosmos ist” probieren: https://www.scienceblogs.de/astrodicticum-simplex/2010/05/der-stoff-aus-dem-der-kosmos-ist-die-komplette-rezension.php Kippenhahn ist aber auch super!

  31. #31 malefue
    29. März 2011

    also, wenn dich der bigbang, die entstehung und die ganze geschichte der theorie bis in die gegenwart interessiert, ist eigentlich “big bang” von simon singh immer die erste empfehlung von mir.
    das hat mich als physik-laie (geisteswissenschaftler) über vieles aufgeklärt und das kann man tatsächlich mit ein bisschen schulwissen verstehen. abgesehen davon ist es auch spannend erzählt und beschreibt schön das wissenschaftliche konzept konkurrierender hypothesen. singh hats sogar geschafft, dass ich mitleid mit fred hoyle empfunden habe nach der lektüre. : D
    für andere themen kommen jetzt sicherlich mehr als genug vorschläge von anderen.

  32. #32 Super Tanooki
    29. März 2011

    @ Bjoern

    Danke! Scheint für mich und für den Anfang, genau das richtige zu sein : ) Direkt mal bestellt.

    Ich habe wirklich riesiges Interesse daran. Wie alles funktioniert und warum etwas ist, wie es ist usw. Ich wills einfach lernen und verstehen

    Müsste ja eigentlich schonmal die richtige Vorraussetzung sein ; )

  33. #33 Hajo Becker
    29. März 2011

    Also trotz der hilfreichen Kommentare von Florian und Bjoern muss ich da doch noch mal nachhaken. Wenn ich wie Edwin Hubble in dem zitierten Artikel die Radialgeschwindigkeit gegen die Entfernung auftrage, habe ich als Steigung die “Hubble-Konstante” in km/s/Mpc. Die Messwerte liegen nur dann auf einer Geraden, wenn die Expansionsgeschwindigkeit konstant ist.
    @Bjoern: was du mit dem zweimaligen “heute” meinst, verstehe ich nicht. Ich kann “heute” doch nur die Geschwindigkeit der n Lichtjahre entfernten Galaxie messen, die sie vor n Jahren hatte (es ist mir bewusst, dass das ziemlich platt ausgedrückt ist, da ja eigtl. nicht die Galaxie wegfliegt, sondern sich der Raum ausdehnt usw.).

  34. #34 Bjoern
    29. März 2011

    @Hajo Becker: Was man eigentlich misst, ist natürlich nicht die Geschwindigkeit selbst, sondern die Rotverschiebung. Letztere ist aber tatsächlich proportional zur “Geschwindigkeit”, welche die Galaxie heute hat – nicht zur Geschwindigkeit zu der Zeit, als das Licht ausgesandt wurde. Ich weiß, ist nicht gerade intuitiv – aber stimmt so tatsächlich! Außer “kommt so aus der Rechnung raus” kann ich dafür leider keine Erklärung anbieten…

  35. #35 Bjoern
    29. März 2011

    @Hajo Becker: Siehe auch hier (gleich die ersten paar Absätze, über dem ersten Diagramm):
    https://www.astro.ucla.edu/~wright/cosmo_02.htm#MD

  36. #36 Bjoern
    29. März 2011

    @Hajo Becker: (zum 3. Mal!) Ups, kleiner Fehler – die Rotverschiebung z ist im Allgemeinen nicht proportional zur heutigen Geschwindigkeit (gilt nur für kleine z!)). Aber man kann beide ineinander umrechnen, indem man ein kosmologisches Modell ansetzt. Die Hauptaussage oben bleibt aber richtig: die heutige Geschwindigkeit ist H mal die heutige Entfernung.

  37. #37 Hajo Becker
    29. März 2011

    @Bjoern: das soll ja jetzt keine Privatdiskussion auf Florians Blog werden, aber ich möchte doch noch mal nachfragen. Erstens: mit H meinst du jeweils “H null” (den heutigen Wert)? Und wo ist die Rechnung, aus der das “so rauskommt”? Ich möchte das schon gerne im Detail nachvollziehen (darf gerne auch “technisch” sein).

  38. #38 Bjoern
    29. März 2011

    @Hajo Becker: Ja, ich meine H0 (den heutigen Wert).

    Die Rechnung ist eigentlich einfach: sei D(t) die Entfernung zur Zeit t, D0 die Entfernung jetzt (t = t0), dann gilt mit dem Skalenparameter a (nach Definition von a): D(t) = D0 * a(t) / a0.

    Leitet man das ab, so erhält man für die Geschwindigkeit: v(t) = D0 * a_dot(t)/a0. Für t = t0 erhält man also v0 = D0 * a_dot0(t)/a0.

    Der letztere Quotient ist aber H0 (nach Definition), also hat man in der Tat: v0 = H * D0.

  39. #39 sdf
    30. März 2011

    Hihi, kaum werden Formeln gebracht, erstickt die Diskussion
    😉

  40. #40 Frank Wappler
    30. März 2011

    Björn Reinhardt schrieb (29.03.11 · 14:24 Uhr):

    > Wie schnell bewegt sich dann eine Galaxie von uns weg, die 4100 Megaparsec entfernt ist?

    Jemand (nennen wir denjenigen “B”), der von uns (“A”) 4100 Megaparsec entfernt ist, bewegt sich nicht “von uns weg“.
    Die Vorgabe eines einvernehmlichen Distanzwertes zwischen A und B, als ein bestimmter Eigenwert des aus A und B bestehenden Systems setzt die (zumindest gedanken-experimentelle) Bedingung voraus,
    dass A und B dabei zueinander ruhten, d.h. dass sich dieser Messwert aus dem entsprechenden Eigenzustand des aus A und B bestehenden Systems ermitteln lässt;
    oder, mit etwas gutem Willen bzw. einer ggf. weniger einvernehmlichen Definition von Entfernung, zumindest die Bedingung,
    dass A und B dabei zueinander starr waren.

    Die Feststellung von Entfernung ist nicht mit einer Feststellung von Bewegung kompatibel, in deren Verlauf sich Entfernung änderte.
    (Im Falle gegenseitiger Starrheit von A und B käme allerdings z.B. “Bewegung quer zur Entfernung” in Frage, z.B. dass “A und B umeinander kreisen”.)

    Wenn man nun einen bestimmten Entfernungswert “zwischen A und B” vorgibt und trotzdem sinnvoll nach einer Bewegung “entlang der Trennungrichtung” fragen will,
    dann sind zwangsläufig dritte Beteiligte “G” involviert;
    d.h. man kann fragen und messen

    – falls A und B zueinander ruhten z.B.
    “Wie groß war die Durchschnittsgeschwindigkeit von G gegenüber (dem System bestehend aus) A und B?”, oder

    – falls A und B (lediglich) zueinander starr waren z.B.
    “Wie groß war die Geschwindigkeit, in Betrachtung der Grenzübergangs, von B und G zueinander als B und G einander passierten?”, oder
    “Wie groß war die Geschwindigkeit, in Betrachtung der Grenzübergangs, von A und G zueinander als A und G einander passierten?”;

    aber sicher nicht:
    “Wie groß war die Geschwindigkeit, in Betrachtung der Grenzübergangs, von A und G zueinander als B und G einander passierten?”.

    In so fern wäre es offenbar eine Milchmädchenrechnung,
    den im Artikel angegebenen Wert
    > 73.8 ± 2.4 Kilometer pro Sekunde pro Megaparsec
    mit 4100 Megaparsec zu multiplizieren und sich einzubilden, dadurch einen bestimmten Wert einer Geschwindigkeit zu erhalten;
    oder z.B. auch “nur” mit 4 Megaparsec.

  41. #41 MartinB
    30. März 2011

    @HajoBecker
    Kleiner Tipp noch zur Veranschaulichung: Die Rotverschiebung, die wir messen, kommt nicht (wie häufig falsch dargestellt, neulich sogar vonmir, schäm) durch den Dopplereffekt zu Stande, sondern dadurch, dass sich die Wellenlänge des Lichts sozusagen “mit der Raumzeit” ausdehnt. Ein Lichtstrahl, der zu einer Zeit ausgesandt wurde, wird deswegen so stark rotverschoben, wie sich der Raum, den er durchquert, in der Zwichenzeit gedehnt hat – das ist aber gleichzeitig auch ein Maß für die Geschwindigkeit, die die weit entfernte Galaxie heute hat.

  42. #42 knorke
    30. März 2011

    Ich habe neulich ne Doku gesehen, die hieß “Das Ende des Universums” oder so ähnlich. Dort ging es um die drei denkbaren Enden des Universums (Big Rip, Big Crunch und halt einfach immer weiter ausdehnen, erkalten und schluss).

    Was ich dort sehr bemerkenswerte fand, war die Aussage eines Wissenschaftlers, die mit dem Thema selbst gar nicht so extrem verknüpft war. Er sagte, dass wir nur Dinge beobachten können, deren Licht seit dem Urknall vor 16 Milliarden Jahren (oder wieviel das war) überhaupt bis zu uns gelangen konnte. Ob es dahinter noch weitere Materie gibt, kann man daher eigentlich gar nicht so genau wissen, wenn ich den Tenor dieser Aussagen richtig deute. Dazu fallen mir zwei Fragen ein:
    1) Könnte es also sein, dass Materie jenseits des für uns sichtbaren Raums exisiert, die quasi die ganze Materie im sichtbaren Bereich mittles Gravitation nach aussen zieht?
    2) Irgendwann hat mir hier mal wer erklärt, dass das Universum endlich ist, was man – vereinfacht erklärt daran merkt, dass es nachts Dunkel ist. Aber nach dem was oben steht kann es durch durchaus sein, dass es jenseits des beobachtbaren Bereich unendlich mit dem Universum weitergeht und wir nur noch nicht das Licht von dort haben ankommen sehen.

    Ich vermute mal, dass die Frage nach der Endlichkeit des Universums ein bissel weniger trivial beantwortbar ist als mit dem Satz “Weils nachts dunkel ist”, aber auf solche Fragen kommt man halt, wenn man solche Aussagen hört.

  43. #43 Florian Freistetter
    30. März 2011
  44. #44 martin
    30. März 2011

    Ich behaupte mal ketzerisch als Laie: Nicht das Universum expandiert, sondern es kollabiert. Es sind nicht die Sterne, die sich unseren Längenmaßen entsprechend fortbewegen, sondern es ist umgekehrt.

    Könnte man diese Behauptung widerlegen?

  45. #45 Findelkind
    30. März 2011

    Ich denke, ich habe das mit dem expandierenden Raum zumindest im Ansatz einigermaßen verstanden.

    FRAGE: Ist die Rotverschiebung der einzige Hinweis, den das Universum hier gibt? Oder gibt es weitere meßbare Größen, die die Sichtweise stützen?

  46. #46 Christian
    30. März 2011

    ist ja alles ganz nett, aber wie realistisch kann so ein theoriengebäude wirklich sein, wenn es mit den entfernungen, und folglich geschwindigkeiten hapert ?

    sind die theorien da so fehlertolerant ? denn realistisch betrachtet, vergrößert sich doch der “entfernungsfehler” ab ein paar 1kpc doch schon sehr stark, zumindest hat man als laie diesen eindruck, wenn man dann und wann so meldungen hört, naja die galaxie xy ist doch um x lj näher oder ferner als man dachte.

    der trick mit den cepheiden ist ja ganz nett und hilfreich, aber was nutz es wenn ich mich bei der entfernungsschätzung zur heimatgalaxie vertue.

    was ist da eigentlich das bindeglied zwischen “parallaxenentfernungsmessung” (geht ja nur ein paar 100 lj) und den nähsten ceph.
    oder hat man da das glück dass noch ceph., parallaxenmäßig einigermaßen genau festgestellt werden können.

  47. #47 Dorian
    30. März 2011

    Was ich mich zur Ausdehnung des Raumes/Universums nur gerade Frage ist folgendes: Die durch die Hubble-Konstante beschriebene Ausdehnung ist ja an jedem Ort im Universum gleich (d.h. egal wo ich mich im Universum befinde, entfernen sich die umliegenden Objekte mit diesen ~74 km/s/Mpc). Warum dehnt sich scheinbar nur der leere Raum zwischen den Galaxien aus? Müsste sich der Raum nicht auch beispielsweise in unserem Sonnensystem ausdehen und so die Entfernungen zwischen Sonne und den Planeten vergrößern? Oder wirkt hier die Gravitation entgegen?

  48. #48 Andreas Müller
    30. März 2011

    Das (lineare) Hubble-Gesetz verliert ab einer Entfernung von ca. 400 Mpc oder z = 0,1 seine Gültigkeit – dann müssen je nach Kosmologie komplexere Gesetze angewendet werden. Siehe: https://www.wissenschaft-online.de/astrowissen/lexdt_h05.html#hubeff
    Bitte vergesst es, die damit zusammenhängenden Geschwindigkeiten ernst zu nehmen und als wahre Geschwindigkeiten zu interpretieren; was man schon daran merkt, dass ab einer bestimmten Entfernung die Fluchtgeschwindigkeit größer wird als die Lichtgeschwindigkeit.

    Nach den aktuellen Messdaten für die Häufigkeit von gewöhnlicher (baryonischer) Materie, Dunkler Materie und Dunkle Energie wird das Universum im konventionellen Standardmodell beschleunigt und ewig expandieren und dabei auskühlen. Das nennt man den “Big Whimper”.
    Siehe: https://www.wissenschaft-online.de/astrowissen/lexdt_b03.html#bw

    Dieses Schicksal wäre nur mit nicht konventionellen Modellen zu umgehen, z.B., falls es räumliche Extradimensionen geben sollte, mit dem “Zyklischen Universum”, was nach heutigem Stand sehr spekulativ ist.
    Siehe: https://www.wissenschaft-online.de/astrowissen/lexdt_z02.html#zyk

    @Findelkind
    Die expandierende Raumzeit wird neben der Rotverschiebung durch folgende weitere Beobachtungen gestützt: 1) die Lichtkurven weit entfernter, explodierender Weißer Zwerge (Supernovae Typ Ia); 2) die großräumige Struktur und deren Wachstum (also die Verteilung von Galaxienhaufen); 3) die Häufigkeit der leichtesten Elemente im Kosmos, die nur durch eine frühe, heiße Phase erklärt werden kann.

    @Christian
    Die Cepheiden-Entfernung ist unabhängig von der Entfernung seiner Heimatgalaxie. Aus der Perioden-Leuchtkraft-Relation folgt aus der beobachteten Periode seine Leuchtkraft, d.h. die absolute Helligkeit M. Eine weitere Beobachtungsgröße ist seine scheinbare Helligkeit m. Aus diesen beiden Größen M und m folgt dann über den Distanzmodul die Entfernung des Cepheids.
    Siehe: https://www.wissenschaft-online.de/astrowissen/lexdt_d02.html#dis

    Beste Grüße,
    Andreas

  49. #49 Bullet
    30. März 2011

    @dorian:
    Kleine Rechnung dazu: 1 Mpc = 3,26 Mio. LJ = 3,1*10^20 km.
    Der entsprechende Wert auf Millionen Kilometer statt Mpc umgerechnet wäre also
    ~74 km / 3,1*10^14 = 2,3 Nanometer pro Sekunde und Millionen Kilometer.
    Reicht dir das als Überschlag bezüglich der Überlagerung durch lokale Gravitationswirkungen??

  50. #50 Bullet
    30. März 2011

    Upsi.

    0,23 nm.

    sorry.

  51. #51 SCHWAR_A
    30. März 2011

    @Bullet:
    Sind das tatsächlich etwa 1.09 m/Jahr/AE ?
    Sowas müßte aber doch mit Radar-Messungen meßbar sein, oder?

  52. #52 Andreas Abendroth
    30. März 2011

    Hallo Florian,

    könnte man dadurch, dass man die Relativbewegung zwischen den Galaxien beobachtet, auf den Ursprungsort der Ausdehnung des Universums, also den Ort des “Urknalls” (Urknall nur in Anführungszeichen gesetzt, da keine bessere/zutreffendere Bezeichnung vorhanden) schließen?

    Andreas

  53. #53 Thomas J
    30. März 2011

    @Andreas

    Ja, kann man… das Ergebnis ist, dass der Knall überall war 🙂

  54. #54 .deb
    30. März 2011

    @Frank Wappler:

    Jemand (nennen wir denjenigen “B”), der von uns (“A”) 4100 Megaparsec entfernt ist, bewegt sich nicht “von uns weg”.
    Die Vorgabe eines einvernehmlichen Distanzwertes zwischen A und B, als ein bestimmter Eigenwert des aus A und B bestehenden Systems setzt die (zumindest gedanken-experimentelle) Bedingung voraus, dass A und B dabei zueinander ruhten, d.h. dass sich dieser Messwert aus dem entsprechenden Eigenzustand des aus A und B bestehenden Systems ermitteln lässt.

    (1) Es gibt bisher keine Vereinheitlichung von Quantenmechanik und ART, insofern ist es unphysikalisch, Begriffe wie “Eigenwert”, “Eigenzustand”, etc. im Zusammenhang mit einem expandierendem Universum zu benutzen.

    (2) Aber selbst wenn wir von einer flachen Raumzeit ausgehen: Physik funktioniert, wenn man die richtigen Tools einsetzt. Für Bewegung von Himmelskörpern ist das dann die klassische Mechanik mit ihren klar definierten Begrifflichkeiten. Natürlich kann man das Ganze quantenmechanisch betrachten, dabei kommt aber aufgrund der winzigen de-Broglie-Wellenlängen nur heraus, dass die klassische Mechanik richtig bleibt. Insofern ist es unsinnig, die kompliziertere Theorie zur Definition von Begrifflichkeiten heranzuziehen.

    (3) Stimmt nichtmal dein Argument. Die Vorstellung einer Distanz setzt nicht vorraus, dass A und B zueinander ruhen. Weder in der klassischen Physik (Zustand charakterisiert durch Punkt im Phasenraum) noch in der Quantenmechanik, wo Distanz und Relativbewegung Differenzen von Orts bzw. Impulsmittelwerten entsprechen. Ortseigenzustände sind ein mathematischer Trick und kein mögliches Messergebnis.

  55. #55 Dorian
    30. März 2011

    @Bullet:
    Also wenn dein Wert stimmt und ich mich jetzt nicht verrechnet habe, dann komme ich auf 10,9 Meter pro Jahr und AE. Müsste das nicht auffallen? Seit Anbeginn der astronomischen Beobachtungen müssten da doch schon einige Meter zusammen gekommen sein?!

  56. #56 Bullet
    30. März 2011

    @SCHWAR_A und Dorian:
    Die Einheit ist Geschwindigkeit pro Entfernung. Bei Umrechnung von nm/s nach m/a komme ich auf 7,25 mm/a auf 10^6 km. Bei einer AE als Referenzentfernung kommt ein Wert von 1,09m heraus. Wenn ich das richtig überblicke, ist eine Entweichgeschwindigkeit von 1 Meter pro Jahr und AE zwischen zwei Objekten, die jeweils 1 und 2 AE von uns entfernt sind und uns gegenüber eigentlich unveränderlich in Ruhe sein sollen, nicht wirklich mit irgendeiner Meßmethode zu erfassen. Noch dazu, wo Radar- oder Lasermessungen eine ziemlich neue Methode sind.
    Jetzt mal ernsthaft: hält es echt jemand für möglich, daß wir die Entfernung zu Alpha Centauri (übergrob gerundet: 250 000 AE) jedes Jahr einmal messen können und jedes Jahr feststellen, daß der Stern sich von unserem Stern 250 km entfernt?

    Hallo? Das Sonnensystem ist ein Staubkorn in einem Wirbelsturm. Und A Cen ist der uns zur Zeit nächstliegende weitere Staubkrümel.

    Wer gibt die Entfernung zwischen Alexanderplatz und Freiheitsstatue in Mikrometern an? Und wie konstant kann dieser Wert auf einem tektonisch aktiven Planeten sein?

  57. #57 Reinhard
    30. März 2011

    tschuldigung – ich denk da zu laienhaft! – Würd gerne wissen obs Hinweise darauf gibt, dass sich die Galaxien auf geraden Bahnen durch den Kosmos bewegen. Wenn es einen Urknall gab, dann sollten sich ja die Galaxien (Materie) davon weg bewegen.
    Es gibt aber Kollisionen von Galaxien miteinander – machen da manche eine Kurve?
    Durch Gavitationskräfte? auch dunkler Materie?

  58. #58 Bullet
    30. März 2011

    @Reinhard:

    Würd gerne wissen obs Hinweise darauf gibt, dass sich die Galaxien auf geraden Bahnen durch den Kosmos bewegen.

    Das tun sie natürlich nicht, denn Galaxien beeinflussen sich gegenseitig durch Gravitation.

    Wenn es einen Urknall gab, dann sollten sich ja die Galaxien (Materie) davon weg bewegen.

    Wieso? Wie schon häufiger erwähnt, bewegen sich laut dem Urknallmodell die Galaxien gar nicht. Es ist der Raum selbst, der sich aufpustet wie ein Luftballon. Das Gleichnis kennst du doch, oder?

  59. #59 Dorian
    30. März 2011

    @Bullet:
    OK, dann habe ich mich irgendwo um eine Zehnerpotenz verrechnet.

    Danke für deine Erklärung. Natürlich denke ich nicht, dass ihr sonst nichts zu tun habt, als ständig irgendwelche Entfernungen zu messen. Dein Vergleich am Ende deines Textes klingt einleuchtend. Ich habe mich eben nur gefragt, ob sich eine überall gleichmäßige Ausdehung des Raumes nicht auch im Laufe der Zeit auf kleinen Skalen bemerkbar machen müsste. Dass ich nicht gleich überblickt habe, dass es sich dabei um verschwindend kleine Bewegungen handelt, sei meinem laienhaften Denken geschuldet.

  60. #60 Andreas Abendroth
    30. März 2011

    @Thomas J: Da vergrößert sich wohl der Raum an sich und die Galaxien stoßen nicht in neue Gebiete vor. Hatte ich mir noch gar nicht überlegt. Danke.

    Andreas

  61. #61 Bullet
    30. März 2011

    @Dorian:
    um Gottes willen, ich wollte nicht mit dem Rohrstock argumentieren. 😉 Aber man macht sich meistens echt keine Vorstellungen davon, wie entsetzlich viel leerer Raum zwischen den Sternen vorhanden ist – und trotzdem braucht man intergalaktische Entfernungen, um genug Δv anzuhäufen, um die Expansion sichtbar zu machen.
    Das ist schon krass viel.

    Ich habe mich eben nur gefragt, ob sich eine überall gleichmäßige Ausdehung des Raumes nicht auch im Laufe der Zeit auf kleinen Skalen bemerkbar machen müsste.

    Na ja … irgendwie schon. Aber dann wird eben “im Laufe der Zeit” ein enormer Faktor. Vielleicht 100 Milliarden Jahre? *g*

  62. #62 Bjoern
    30. März 2011

    @martin: Du drückst dich etwas unklar aus – ich nehme an, du meinst, unsere Längenmaßstäbe werden immer kleiner, und deshalb erscheint es uns, als ob das Universum sich ausdehnen würde? Dieses Hypothese wird schon alleine dadurch widerlegt, dass wir eben nicht überall Ausdehnung beobachten (z. B. nicht auf der Erde, im Sonnensystem usw.), sondern nur auf großen Skalen (so etwa ab 10 Megaparsec).

  63. #63 Bjoern
    30. März 2011

    @Findelkind: Für die Urknall-Theorie als Ganzes gibt es eine ganze Menge Hinweise – und da diese Theorie die Ausdehnung des Raumes vorhersagt, gibt es also auch indirekt eine ganze Menge Hinweise auf die Ausdehnung des Raumes. Ich hab’ vor Jahren da mal einiges zusammen gesucht:
    https://www.feuerbachers-matheseite.de/indexd.htm

  64. #64 Bjoern
    30. März 2011

    @Christian: Die hauptsächliche Methode, die zwischen Parallaxenmessungen und Cepheidenbeobachtungen benutzt wird, ist das Hertzsprung-Russell-Diagramm. Inzwischen wurden aber auch bereits (vom HIPPARCOS-Satelliten) die Parallaxen einiger Cepheiden direkt gemessen.

  65. #65 Bjoern
    30. März 2011

    @Dorian: Zusätzlich zu Bullets Argument gilt übrigens tatsächlich (wie du selbst schon vermutest hattest), dass auf “kleinen” Skalen (unser Sonnensystem usw.) die lokale Gravitation überwiegt.

  66. #66 Findelkind
    30. März 2011

    @Bjoern: Danke, ich werde das durcharbeiten.

    Ich bin zwar wissenschaftlich arg vorbelastet, aber eben nur ein Amateur auf diesem Gebiet. Rechnet in Zukunft mit weiteren Fragen…

    Gemeinsam mit einem Kollegen (ich bin Grafiker und 3D-Animator) bereite ich eine Art Remake von “Powers of ten” vor. Sowas gibt es schon, ist mir bekannt, aber das geht noch sicher aktueller und besser.

    Ich weiss zwar schon einiges, da mich diese Thematik schon seit meiner Kindheit nicht mehr loslässt, aber es gibt ja nie einen Grund, sein Wissen nicht zu erweitern…

  67. #67 Frank Wappler
    30. März 2011

    .deb schrieb (30.03.11 · 13:42 Uhr):

    > (1) Es gibt bisher keine Vereinheitlichung von Quantenmechanik und ART, insofern ist es unphysikalisch, Begriffe wie “Eigenwert”, “Eigenzustand”, etc. im Zusammenhang mit einem expandierendem Universum zu benutzen.

    Ich weiß zwar nicht, was “Vereinheitlichung von Quantenmechanik und ART” bedeuten soll, oder woher die Wweisheit stammt, dass es diese nicht gäbe;
    aber ich habe die Begriffe “Eigenwert”, “Eigenzustand”, etc. schlicht auf ein System mit benannten gegenseitig beobachtbaren und unterscheidbaren Beteiligten (“A”, “B”, …) bezogen bzw. auf den entsprechenden Messwert (“sie ruhten zueinander”, oder zumindest “sie waren starr zueinander”) bezogen, der durch Anwendung des entprechenden nachvollziehbaren Messoperators ggf. einvernehmlich herauszufinden wäre.
    Wenn das unphysikalisch ist, dann bin ich kein Physiker.

    > (2) Aber selbst wenn wir von einer flachen Raumzeit ausgehen:

    Wir müssen uns nicht auf die Betrachtung von Versuchen beschränken, in denen A und B und (nach J. Synge mindestens drei) weitere geeignete (d.h. beobachtbare und unterscheidbare) Beteiligte zueinander ruhten, also starr und flach waren;
    schließlich hatte ich oben ausdrücklich den separaten Fall betrachtet, dass A und B (nur) zueinander starr waren.
    Aber gewiss können wir diesen Spezialfall (gegenseitige Ruhe) betrachten.

    > Physik funktioniert, wenn man die richtigen Tools einsetzt.

    Auffallend richtig. Und diese richtigen Tools sind: nachvollziehbare Gedankenexperimente, in Anwendung auf (tatsächliche oder hypothetische) Beobachtungsdaten.

    > Für Bewegung von Himmelskörpern ist das dann die klassische Mechanik mit ihren klar definierten Begrifflichkeiten.

    Wie kommst du darauf, dass es die klassische Mechanik über irgendwelche klar definierte Begrifflichkeiten verfüge?
    Welche gedanken-experimentelle Messoperation läge z.B. dem (offenbar in der klassischen Mechanik auftretenden) Begriff “gleichmäßig bewegt” zugrunde??

    > winzige[…] de-Broglie-Wellenlängen
    > Impulsmittelwerte[…]
    > Zustand charakterisiert […] im Phasenraum

    Mein vorausgegangener Kommentar (30.03.11 · 03:37 Uhr) betraf allein Geometrie und
    Kinematik; nicht Dynamik.
    Natürlich: falls von zwei Beteiligten in einem bestimmten Versuch gefunden wurde, dass sie zueinander ruhten, dann ließe sich in diesem Versuch kein Messwert zum dazu konjugierten Operator (“Impuls in Bezug aufeinander”) angeben.

    > Ortseigenzustände sind ein mathematischer Trick und kein mögliches Messergebnis.

    Ich weiß zwar nicht, was “Ortseigenzustände” bedeuten sollen;
    es ging lediglich um einzelne gegenseitig beobachtbare und unterscheidbare Beteiligte und deren zu messende geometrische (bzw. betreffs des Beteiligten “G” kinematische) Beziehungen zueinander.

    Von diesen Beteiligten als “Orten” zu denken ist i.A. nicht hilfreich, da sich verschiedene Beteiligte auch treffen und passieren könnten (wie B und G im obigen Beispiel).

    Zum Thema “Messung geometrischer Beziehungen” (im Rahmen der ART) sind die Beiträge von J. L. Synge (Stichwort “chronometric geometry”) recht nachvollziehbar und empfehlenswert …

  68. #68 Bjoern
    30. März 2011

    @Findelkind:

    Gemeinsam mit einem Kollegen (ich bin Grafiker und 3D-Animator) bereite ich eine Art Remake von “Powers of ten” vor. Sowas gibt es schon, ist mir bekannt, aber das geht noch sicher aktueller und besser.

    Tolle Idee! Ich stelle mich gerne als wissenschaftlichen Berater zur Verfügung… 😉

  69. #69 Bullet
    30. März 2011

    @Wappler:

    Auffallend richtig. Und diese richtigen Tools sind: nachvollziehbare Gedankenexperimente, in Anwendung auf (tatsächliche oder hypothetische) Beobachtungsdaten.

    Du hast was vergessen:
    Beschränkung der Komplexität auf das unerläßliche. Weniger ist mehr.
    Uvm.
    Merkste wat?

  70. #70 Niels
    30. März 2011

    @Frank Wappler

    Und diese richtigen Tools sind: nachvollziehbare Gedankenexperimente, in Anwendung auf (tatsächliche oder hypothetische) Beobachtungsdaten.

    Klare, verständliche Sprache und der Verzicht auch unnötige Schwurbeleien, nur um das eigene Ego zu streicheln, sind ebenfalls mächtige Werkzeuge der Naturwissenschaften.

  71. #71 Bjoern
    30. März 2011

    @Frank Wappler:

    Ich weiß zwar nicht, was “Vereinheitlichung von Quantenmechanik und ART” bedeuten soll, oder woher die Wweisheit stammt, dass es diese nicht gäbe;…

    Weisst du das echt nicht, oder stellst du dich nur mal wieder absichtlich blöd?
    https://de.wikipedia.org/wiki/Quantengravitation

    …aber ich habe die Begriffe “Eigenwert”, “Eigenzustand”, etc. schlicht auf ein System mit benannten gegenseitig beobachtbaren und unterscheidbaren Beteiligten (“A”, “B”, …) bezogen bzw. auf den entsprechenden Messwert (“sie ruhten zueinander”, oder zumindest “sie waren starr zueinander”) bezogen, der durch Anwendung des entprechenden nachvollziehbaren Messoperators ggf. einvernehmlich herauszufinden wäre.

    Na, da bin ich mal gespannt, ob du nur daherreden kannst oder tatsächlich weisst, von was du redest… Bitte gib’ doch mal an, wie der “Messoperator” für die beiden genannten “Messwerte” jeweils aussehen müsste!

    Wie kommst du darauf, dass es die klassische Mechanik über irgendwelche klar definierte Begrifflichkeiten verfüge?

    Weil das halt so ist? Siehe diverse Einführungsbücher zur Mechanik; besonders klar war, soweit ich mich erinnere, die Darstellung im Buch von Landau und Lifschitz.

    Welche gedanken-experimentelle Messoperation läge z.B. dem (offenbar in der klassischen Mechanik auftretenden) Begriff “gleichmäßig bewegt” zugrunde??

    Eine Bewegung ist gleichmäßig, wenn in gleichen Zeitabständen jeweils gleiche Strecken zurück gelegt werden. Und die Messung von Zeitabständen und Strecken ist (im Rahmen der klassischen Mechanik) für jeden offensichtlich – nur für dich anscheinend nicht…

    Natürlich: falls von zwei Beteiligten in einem bestimmten Versuch gefunden wurde, dass sie zueinander ruhten, dann ließe sich in diesem Versuch kein Messwert zum dazu konjugierten Operator (“Impuls in Bezug aufeinander”) angeben.

    Wie vermutet: du weisst tatsächlich nicht, von was du redest. Der zum Operator, welcher “Ruhe / Bewegung relativ zueinander” misst, konjugierte Operator ist nicht der Impulsoperator – wie jeder, der auch nur die elementarsten Grundzüge der Quantentheorie kennt, weiss.

    Ich weiß zwar nicht, was “Ortseigenzustände” bedeuten sollen;…

    Äh, selbstverständlich die Eigenzustände der Ortsoperators? Sag’ jetzt bloss nicht, den Operator kennst du nicht…?

    Wenn das unphysikalisch ist, dann bin ich kein Physiker.

    Nichts von dem, was du hier geschrieben hast, lässt darauf schließen, dass du ein Physiker bist. Es klingt eher so, als hättest du zwar viele physikalische Bücher gelesen, aber das meiste davon missverstanden…

  72. #72 Bullet
    30. März 2011

    @Niels:
    *high five*

  73. #73 Findelkind
    30. März 2011

    @Bjoern

    Tolle Idee! Ich stelle mich gerne als wissenschaftlichen Berater zur Verfügung… 😉

    Da werde ich eventuell drauf zurückkommen. Kann allerdings noch eine Weile dauern, da es sich um ein typisches, nicht kommerzielles Feierabendprojekt handelt, das noch ganz am Anfang steht.

    Aber schonmal vielen Dank für das Angebot…

  74. #74 Niels
    30. März 2011

    @Bullet
    *high five* 😉

  75. #75 endre kereszturi
    25. Oktober 2011

    Es könnte bei der Grundfrage – der Wert der Hubble-Konstante – entscheidend behilflich sein, wenn man mein folgendes Forschungsergebnis unvoreingenommen in Betracht zieht. Es handelt sich dabei um eine Zusammenhang zwischen dem “Weltalter” (hier einmal mit t_U = 13,75 Gj plusminus 0,04 Gj Messfehler berücksichtigt) und einigen mikrophysikalischen Naturkonstanten.

    Ausgangspunkt ist in meiner “RELATIVISTISCHEN METROLOGIE” der Lorentz-invariante Ausdruck (1 Meter x 1 Sekunde) – siehe https://www.naturkonstanten.info

    h-quer/c/t_U = (1 Meter x 1 Sekunde) x sqr(m_Z x m_W) = 1,523… e-25 kg m s.

    Hier zeichnen neben den üblichen h und c Konstanten m_Z und m_W die Ruhemassen der elektroschwachen Feldbosonen “Z” und “W”. Und die linke Seite verbirgt einen Hinweis auf die mysteriöse kosmische Beschleunigung 2Pi x c/t_U = 4,35… e-9 ms^-2.

    Eventuell lässt es sich zeigen, dass der Massenwert 1,523… e-25 kg mit “virtueller Higgs-Bosonen” etwas zu tun – ich arbeite daran…

    Die obige Gleichung wurde aus der Vermutung abgeleitet, dass die “dunkle Energie” mit c^5 x t_U/G (G als Gravitationskonstante) gleichzusetzen ist. — Grüsse: E. K.

  76. #76 Florian Freistetter
    25. Oktober 2011

    @kereszturi: “mein folgendes Forschungsergebnis unvoreingenommen in Betracht zieht”

    “Forschungsergebnisse” werden normalerweise nur dann auf privaten Homepages veröffentlicht, wenn sie zu falsch sind, um in echten wissenschaftlichen Zeitschriften publiziert zu werden…

  77. #77 Wurgl
    25. Oktober 2011

    Klingt nach Radosophie.

  78. #78 Oliver Debus
    25. Oktober 2011

    Also eines macht mich nachdenklich. Wenn es in Holland 15 Millionen Einwohner hat, es dort aber 16 Millionen Fahrräder gibt, stellt sich mir die Frage, warum es eine Millionen Fahrräder mehr gibt als Einwohner? Kann es sein, dass diese Fahrräder Ausserirdischen gehören, die sie in Holland geparkt haben und so den Holländern geheime Informationen über das Universum hinterlassen haben?
    Fragen über Fragen…. 😉

  79. #79 HaDi
    25. Oktober 2011

    Oha, den Besucher hatten wir schon mal.

  80. #80 rolak
    25. Oktober 2011

    mal‘? ^^

  81. #81 endre kereszturi
    25. Oktober 2011

    Es freut mich wieder mit alten Freunden zu diskutieren… Haben Sie, meine Herren, inzwischen etwas Neues hervorgebracht? Der Vorwurf “Radosophie” klingt etwas blass – um mich sehr mild auszudrücken – es stimmen nämlich bei mir nicht nur die Zahlenwerte, sondern auch die dazugehörigen physikalischen Dimensionen… Was wäre mit einer Kontrolle bevor jemand sich so blamiert? (Um Nachzudenden aufzufordern wäre ja ganz sicher zu viel…) Grüsse weiterhin: E.K.

  82. #82 Adent
    25. Oktober 2011

    @endre …
    Was ist denn mit ihrem Doktor passiert Herr Kereszturi? Irgendwo unterwegs verloren gegangen oder britisches Understatement? Nach einem kurzen Besuch auf ihrer Homepage nebst Jesus Horoskop frage ich mic hwas sie so im Laufe ihres sicherlich reichen wissenschaftlichen Lebens veröffentlicht haben. Damit meine ich nicht irgendwelche obskuren Kommentare oder pseudowissenschaftlichen Bücher sondern wissenschaftliche Literatur. Sie wissen schon, die mit Impact-Faktor usw.
    MfG Dr. rer. nat. Adent

  83. #83 stillerleser
    25. Oktober 2011

    Potzblitz, der Herr Kereszturi ist wieder da. Das hätte ich niiiie gedacht, dass er hier noch einmal einen Kommentar schreibt.
    Björn hatte hier:
    https://www.scienceblogs.de/astrodicticum-simplex/2010/03/albert-einstein-und-die-astrologie.php
    eine lange und intensive Diskussion mit Ihnen wo er Ihnen die Fehler in Ihrer Radosophie vorgerechnet hat. Daraufhin haben Sie sich nicht mehr getraut die Diskussion weiterzuführen. Und nun kommen Sie zu einem anderen Thema wieder als wenn nichts gewesen wäre??

    Hier noch einmal die Fragen vom 17.5.11 aus obigem Thread (Sie wissen ja worum es geht):

    @EK: Haaallooo, Herr Kereszturi! Hööören Sie mich?
    Ich würde immer noch gerne wissen, ob das Ergebnis Ihrer Formel (1) bei Verwendung der Einheiten ft und a (und meinetwegen auch nkg) denn nun “selbstverständlich 1 m * 1 s” oder “selbstverständlich 1 ft * 1 a” ist… und auch, warum sie beides jeweils mehrere Tage lang standhaft behauptet haben. Und warum Sie nicht zugeben können, dass Sie sich eklatant selbst widersprochen haben.

  84. #84 endre kereszturi
    26. Oktober 2011

    WAS geschah früher? Ich habe meinen Diskussionspartner in eine Situation manövriert, wo er dann selber dafür argumentieren musste, was ich ursprünglich – UND AUCH HEUTE – behauptet habe, nämlich für die TATSACHE, dass die Lorentz-invariante metrologische Einheiten-Kombination (1 Meter x 1 Sekunde) immer und überall Lorentz-invariant bleibt, unabhängig davon, welche Masseinheiten jemand gewählt hat. Dies ist ein bewährtes didaktisches Vorgehen – seit “der Griechen”… Niemand konnte bis jetzt mein “AXIOMA PHYSICA HUNGARICA” wiederlegen! Man muss auch den früheren Diskussionsbeitrag VOLLSTÄNDIG FERTIGLESEN: dort findet man auch meine abschliessende Antworten auf die unwissenschaftlichen Vorwürfe.

    Keine Sorgen: mein Doktortitel ist gut aufbewahrt – und natürlich bin ich sehr-sehr beeindruckt auch von “Dr. rer. nat.” etc. – Qualifikationen… (:-) Hoffentlich ist man mit solchen Titeln auch fähig, meine EINFACHE GLEICHUNGEN zu prüfen. Was wäre ZUERST damit, bevor man meine Persönlichkeit disqualifizieren versucht??? Ich selber prüfe sorgfältig alle Reaktionen auf meine Thesen – und was die “offiziellen” Publikationen betrifft, halte mit Gauss: “Wenig aber ausgereift!” (originell lateinisch – Dr. med. Kereszturi Endre)

    Ach, und noch das Buch “Das wahre Jesus-Horoskop”… Wie schon gesagt, ein Horoskop-Mandala ist eine uralte Kombination von “Hier und Jetzt” also von “Raumzeit”! (Hinweis bei Jürgen NEFFE: Einstein. Eine Biographie – 2005 – ODER ANDERSWO? (:-)) DAS IST EINE TATSACHE! Daran ändert nicht, wenn man die Astrologie ständig als “pseudowissenschaftlich” beschimpft. Es ist eben noch nicht “ausgereift”. Aber kulturwissenschaftlich ist nicht mehr ausser Acht zu lassen! (Siehe Kocku von Stuckrad: Das Ringen um die Astrologie – 2000) – Übrigens, seit unserer Publikation plädieren immer mehr Historiker dafür, dass die ominöse Kreuzigung von Jesus von Nazareth im Frühjahr 36. n. Chr. geschah – und nicht irgendwann früher.

    Noch fragen, meine Herren? Oder wollen Sie auch noch über die Euro-Krise mit mir diskutieren – HOCHWISSENSCHAFTLICH?… Grüsse: Dr. E. K. (für alle!)

  85. #85 Florian Freistetter
    26. Oktober 2011

    @kereszturi: “Oder wollen Sie auch noch über die Euro-Krise mit mir diskutieren “

    Ich will gar nicht mit ihnen diskutieren. Selbsternannte Physikrevolutionäre gibt es im Internet haufenweise und auch hier im Blog tauchen sie immer wieder auf (Parallel im anderen Thread erklärt z.B. gerade ein Herr Derksen, warum er klüger als Einstein und der Rest der wissenschaftlichen Welt ist). Trotz all der Genialität schaffen all diese neuen Einsteins es allerdings nie, ihre Erkenntnisse wissenschaftlich korrekt aufzuschreiben, von qualifizierten Fachleuten begutachten zu lassen und sie in einer Fachzeitschrift zu veröffentlichen. Was ist los, Herr Kereszturi? Trauen sie sich nicht? Wissen sie insgeheim, dass ihre Theorie Unsinn ist und wollen sie sie deswegen nicht publizieren? Und kommen sie mir nicht mit “Meine Theorie ist so revolutionär, das würden die etablierten Physiker nie veröffentlichen, das wird alles unterdrückt!!!”. Wenn ihre Forschung vernünftig durchgeführt wurde, können sie sie auch veröffentlichen – selbst wenn sie alles bekannte auf den Kopf stellen würde (sieht man ja gerade gut an den OPERA-Neutrinomessungen). Also: wenn sie den genialen Wissenschaftler nicht nur spielen wollen, dann veröffentlichen sie ihre Forschung! Wenn sie lieber weiter den Physik-Märtyrer und geheimen Revoluzzer spielen wollen, dann ist das ihre Sache. Aber erwarten sie nicht, dass ich sie ernst nehme.

  86. #86 Bullet
    26. Oktober 2011

    Ich habe meinen Diskussionspartner in eine Situation manövriert, wo er dann selber dafür argumentieren musste, was ich ursprünglich – UND AUCH HEUTE – behauptet habe, nämlich für die TATSACHE, dass die Lorentz-invariante metrologische Einheiten-Kombination (1 Meter x 1 Sekunde) immer und überall Lorentz-invariant bleibt, unabhängig davon, welche Masseinheiten jemand gewählt hat.

    Wuha. Klingt nach RICHTIGSTELLUNG!
    Prost.

  87. #87 Adent
    26. Oktober 2011

    @kereszturi
    Ich fragte mich lediglich warum Sie früher als Dr. Endre trallala auftraten und jetzt nur noch als Endre … Es gab hier schon so den einen oder anderen Fall wo “plötzlich” ein Doktortitel verschwand. Das hat nichts mit Herabsetzung des Gegenüber, sondern ganz im Gegenteil mit Heraufsetzung des eigenen Ichs durch illegale Verwendung eines Doktortitels zu tun. Deshalb verschwinden die dann auch schnell wieder bevor sie strafrechtlich belangt werden können. In Ihrem Fall Herr Kereszturi ist das anscheinend nicht gegeben, dem allein diente meine Frage.
    Andererseits ist es schon erstaunlich wie sie hoffen können nach einer wochenlangen Diskussion mit Björn, die sich hauptsächlich durch wirres Durcheinanderwerfen ihrerseits was sie wann und wo behauptet hatten auszeichnete, hier wieder aufzutauchen und erneut den Gleichen schon längst wiederlegten Unsinn abzugeben. Sie haben es nicht verstanden oder, Sie Herr Kereszturi wurden von Björn nicht nur wiederlegt sondern teilweise vorgeführt, dafür noch einmal meinen Dank an ihn!
    Also antworten Sie bitte erstmal auf die schon vor einem halben Jahr erfolgten Fragen (ohne sich wieder zu verlaufen), es besteht ja immer die Hoffnung, daß sie inzwischen die RT ansatzweise verstanden haben.
    Zugegeben, diese Hoffnung ist nach Studium ihrer vorhergehenden Ergüsse nicht sehr groß, aber immer noch um mehrere Zehnerpotenzen größer als die Wahrheit ihrer unwiederlegten “AXIOMA PHYSICA HUNGARICA”.
    MfG Adent

  88. #88 Adent
    26. Oktober 2011

    Im übrigen möchte ich mich für die wiederholte Verwendung des Wortes wiederlegen entschuldigen. Es kam mir gleich komisch vor, aber da Herr Kereszturi es auch verwendet hatte, zweifelte ich natürlich nicht daran, ein kapitaler Fehler.

    Es muß natürlich widerlegen heißen, mea culpa!

  89. #89 endre kereszturi
    26. Oktober 2011

    Es amüsiert mich ausserordentlich, dass hier, WO EIGENTLICH DIE ZAHLEN UND FAKTEN sprechen sollten, nur leere Wortsalaten “verkündet” werden…

    Bei mir ist natürlich alles dokumentiert und eines Tages werden meine Resultaten selbstverständlich auch bei der zuständigen Koryphäer der Physikwissenschaften vorgelegt. Momentan arbeite ich noch an der richtigen Form der späteren Veröffentlichung. DIE ZEIT (Karios) IST NOCH NICHT REIF DAFÜR.

    Aber was ich schon veröffentlicht habe, dafür kann ich jede Zeit gerade stehen. Und so frage ich nochmals: WER BEHAUPTET, DASS DIE AXIOMA PHYSICA HUNGARICA FEHLERHAFT IST? Dass das nicht Lorentz-invariante Kombinationen von Naturkonstanten postuliert? Und dass seine metrologische Grundlage – (1 Meter x 1 Sekunde) – nicht ebenso Lorentz-invariant ist?

    Prüfen Sie doch einmal meinen Brief an die Eöt-Wash-Gruppe (ebenso bei https://www.naturkonstanten.info zu finden ) – Sie finden sicher auch dort etwas “Pseudowissenschaftliches” – nur Schade, dass die neuesten CODATA-Werte von “Big G” seit Jahren mich und meine Theorie bestätigen. Aber wer weiss…

    FAZIT: Blöde Tiraden interressieren mich weiterhin nicht am gerinsten – in der Zukunft reagiere nur noch auf SACHLICHE UND EXAKTE Berechnungen und Bemerkungen. Ansonsten alles Gute und viel Glück! Grüsse: Dr. med. E.K.

  90. #90 Florian Freistetter
    26. Oktober 2011

    @kereszturi: “Momentan arbeite ich noch an der richtigen Form der späteren Veröffentlichung.”

    Eine bessere Ausrede ist ihnen nicht EINGEFALLEN. Schwache LEISTUNG HERR Kereszturi.

    “WER BEHAUPTET, DASS DIE AXIOMA PHYSICA HUNGARICA FEHLERHAFT IST?”

    WARUM SCHREIEN SIE DAUERND RUM?? Sie wollen ernst genommen werden? Dann benehmen sie sich WIE EIN echter Wissenschaftler. Die Taktik: “ALLES was ich sage ist richtig, BIS JEMAND es offiziell widerlegt hat”, ist zwar BEQUEM für sie. Hat aber NICHTS mit Wissenschaft zu tun (Hey, diese irratische groß/klein-Schreibung macht Spaß 😉 ).

    Abgesehen davon behaupte ich, dass ein Werk dem der Autor selbst so einen pompösen Namen gegeben hat, prinzipiell nicht ernst genommen werden muss 😉 Haben sie eigentlich auch schon irgendein Zeugs nach sich selbst benannt?

  91. #91 Adent
    26. Oktober 2011

    @Kereszturi
    Wie schön, daß gerade sie Zahlen und Fakten ansprechen, wo ist denn nun ihr Beleg für die ETNAK zu finden. Ich habe verzweifelt auf ihrer Homepage gesucht dort einen wüsten Zahlensalat nebst Esodressing gefunden, sonst nichts. Mein lieber Herr K, SIE BEHAUPTEN, SIE HÄTTEN bereits vor Jahren, ach was Jahrzehnten (oder waren es gar Jahrhunderte?) die Codata Werte vorab errechnet. WARUM findet sich dann nichts davon in der wissenschaftlichen Literatur?
    Ich vermute etwas und frage sie daher, ist die von Ihnen entwickelte ETNAK Spiegelbild symmetrisch gültig oder nur in einer Symmetriebene?

    Wissen Sie was, ich habe SÄMTLICHE Lottozahlen der letzten 53 Jahre richtig vorhergesagt (den Beweis dafür finden sie in meiner NUMERICA DE LA LOTTO PREDICTA CUM NONSENSIS). Diese werde ich beizeiten veröffentlichen. Bis es soweit ist halte ich mich aber zurück und spiele auch kein Lotto, ich will den anderen, die es nötiger haben als ich ja nichts wegnehmen.
    MERKEN SIE WAS? Ich kann das auch und es macht tatsächlich SPASS so ohne Sinn und Verstand im Blog rumzugröllen wie Herr Dr. K.
    Mal ganz allgemein gesagt, nicht nur an Sie Herr Kereszturi gerichtet sondern auch an alle anderen Gehirnschwurbler (wie z.B. Martin Termin, Hartwig Thim, Dieter B und Dieter B. sowie den Wabbler und weitere Konsorten):
    Es ist die Attitüde mit der Sie hier auftreten, die ganz allgemein auf Ablehnung stößt, nicht ihre Theorien, diese könnte man ja noch versuchen ernsthaft zu diskutieren, aber allein schon der Anfang wenn sie hier auftreten.
    Das läuft in der Regel folgendermaßen ab:

    1. Ich bin derjenige der Bescheid weiß und ihr anderen seid alles nur Knechte des Establishments der Naturwissenschaften und Medizin, die eh völlig falsch liegen und das schon seit Jahrzehnten.
    2. Es interessiert mich gar nicht, was ihr gegen meine Theorie habt, ich weiß ich habe Recht und ihr wollt das nicht anerkennen.
    3. Der wahre Schotte ohne Ende
    4. Dunning-Turner ohne Ende
    5. Immunisierungssyndrom ohne Ende
    6. Selektives Lesen der Argumente anderer, ohne die eigenen Argumente jemals einer Überprüfung zu unterziehen und behaupten des genauen Gegenteils.
    7. Verwirrung in den Grundbegriffen der eigenen Theorie. usw. usw.

    Wie bitteschön Herr K. soll man mit solchen Leuten vernünftig argumentieren?

    Ich gebe zu, sie sind nicht ganz so schlimm wie Name auf Verlangen entfernt, der ist zu all dem oben genannten auch noch süffisant eklig und unterstellt anderen sie würden nicht annähernd auf seinem Niveau argumentieren. Dies stimmt allerdings, so tief herabsinken wie Herr MT kann hier kaum jemand.

    So, das mußte mal raus, ich denke sie werden es ignorieren, genauso wie sie alle anderen Versuche mit Ihnen zu diskutieren bisher mit dem Verweis auf “RECHNEN SIE ERSTMAL” ignoriert haben.
    Ich sage dazu nur RECHNEN SIE NICHT DAMIT 😉

  92. #92 Bullet
    26. Oktober 2011

    Blöde Tiraden interressieren mich weiterhin nicht am gerinsten – in der Zukunft reagiere nur noch auf SACHLICHE UND EXAKTE Berechnungen und Bemerkungen.

    Ja, das ist die Crux mit den ach so von sich überzeugten Revolutionären: sobald sie Kritik nicht in der Form bekommen, mit der sie umgehen können, stecken sie den Kopf in den Sand und verschanzen sich hinter den bremerschen GROSSBUCHSTABEN. Das hatten wir hier alles schon dutzendfach. Wenn wir schon dabei sind: woran erkennt man, daß DIE ZEIT REIF IST? Da kannst du doch bestimmt eine EXAKTE und SACHLICHE Herleitung angeben, oder?

  93. #93 noch'n Flo
    26. Oktober 2011

    Mir sind ja ärztliche Kollegen nicht so oft so richtig peinlich. Aber dieser Fall ist eine eindeutige Ausnahme.

  94. #94 endre kereszturi
    27. Oktober 2011

    Ja, das ist wirklich sehr peinlich, lieber Herr Kollege – aber nicht wegen meiner Person… Es ist ganz klar, dass dieser “Widerstand” von Seiten hoch(?)intelligenter Leuten nur tiefenpsychologisch zu deuten ist. C. G. Jung hat ja mehrmals ausführlich erklärt, dass dort, wo man eine Weigerung zur sachlichen Diskussion findet, MAN IMMER EINE TIEFE – NICHT EINGESTANDENE, UNBEWUSSTE – UNSICHERHEIT DES EIGENEN STANDPUNKTES ANNEHMEN MUSS.

    So ist es auch diesmal der Fall. Man verlangt von mir, dass ich die Grundlagen meiner Theorie “ableiten” soll – es ist aber eben das genuine Wesen der axiomatischen Methode, dass ein Axiom nicht “abgeleitet” werden kann. ES BESTEHT ABER IMMER DIE MÖGLICHKEIT DAS ZU WIEDERLEGEN – ansonsten wäre das nicht wissenschaflich akzeptabel (K. Popper). Nun sollte man also EINFACH zeigen, dass mein Axiom numerisch-rechnerisch-dimensional nicht stimmt. Was geschiet aber? Gerade die von mir angegebenen Werte von der Gravitationskonstante und von der GESAMTMASSE des Sonnensystems werden durch neuesten hochpräzisen Messungen bestätigt.

    Es kann wirklich nur tiefenpsychologisch verstanden werden (ich bemühe mich ständig darum… (:-)), dass diese TATSACHEN Gimnasiasten ohne weiteres “verstehen” , aber die Herren Professoren sich konsequent weigern die metrologischeN Konsequenzen davon zur Kenntnis zu nehmen… Was habe ich mit meiner neuen Gleichung neuerlich UNGEHEUERLICHES gesagt?:

    1./ Die Astronomen haben haben tolle Arbeit geleistet, weil sie den Wert von “Weltalter” des Universums (alias reziprok Hubble-Konstante) mit circa 13,75 Gj erforscht haben.

    2./ Die Schöpfer der elektroschwachen Theorie, und die Experrimentatoren waren ebenso geniell erfolgreich, als sie uns die Existenz und die Ruhemasse-Messwerte von W – und Z- Feldbosonen mit einer ebenso lobenswerten Genauigkeit lieferten.

    Nun, bin ich vom Teufel, wenn ich darauf hingewiesen habe, dass der Quotient sqr(m_Z x m_W)/t_U numerisch wie dimensional mit h-quer/(c x 1Meter x 1Sekunde) ist?

    Und weil ich behaupte, dass diese Zusammenhänge in Rahmen meiner ETNAK-Theorie “relativistisch-metrologisch” zu deuten sind, WEIL der Ausdruck h-quer/(c x 1Meter x 1Sekunde) ebenso LORENTZ-INVARIANT ist, wie (1 Meter x 1 Sekunde) – und so natürlich auch der Ausdruck sqr(m_Z x m_W)/t_U ???

    Lieber Herr Kollege, diesen Fall werden noch auch andere “Begutachter” bewerten – und darin auch IHRE “FACHMEINUNG”! Ich sorge dafür!

    Nun habe ich alles gesagt, was ich momentan zu sagen habe – Ihnen allen, meine hochverewhrte Herren Professoren weiterhin alles Gute – es ist zum Teil tragisch, zum Teil lächerlich, wenn man Leute begegnet, die behaupten, dass sie schon die “volle Wahrheit” besitzen… Bis jetzt denkte ich, dass so etwas eher gewisse Theologen auszeichnet – sie haben mich über etwas Besseres belehrt – herzlichen Dank! Mit meiner Freundlichkeit können Sie selbstverständlich weiterhin RECHNEN – auch wenn ich nicht weiter damit RECHNE, dass Sie auch RECHNEN bereit sind, wenn es um eine einfache Gleichung geht… Aber “vom Bauch heraus” kann man nun mal nicht einmal die formelle Richtigkeit einer Gleichung beurteilen! Und ohne dieser ersten Schritt gibt es in der exakten Wissenschaften keine sinnvolle Diskussion!
    Mit “therapeutisch” freundlichen Grüssen: Dr. med. Kereszturi Endre

    PS.: Wenn Sie Schwierigkeiten haben mit der Aussprache meiner Name, denken Sie an Franz Liszt – oder sagen Sie einfach “Hase”… (:-) E.K.

  95. #95 Adent
    27. Oktober 2011

    Schwallala und Schwurbeli dazu weiterhin die Weigerung auch nur einfachste Fragen zu beantworten, stattdessen tiefenpsychologische Deutungen des letzten Jahrtausends, Respekt Herr Hase was anderes habe ich nicht erwartet. Herr K. ihre RECHNUNGEN wurden bereits widerlegt, wieder und wieder und wieder aber sie weigern sich standhaft dies zu aktzeptieren, was soll man da tiefenpsychologisch hineindeuten?
    Ich warte gespannt auf ihre erste peer-reviewed Veröffentlichung, bis dahin:
    Tschüß wie man in Hamburg sagt.

  96. #96 Bullet
    27. Oktober 2011

    Weia.

    es ist zum Teil tragisch, zum Teil lächerlich, wenn man Leute begegnet, die behaupten, dass sie schon die “volle Wahrheit” besitzen…

    Dieser Satz ist so durchaus brauchbar. Ich denke beim Lesen dieses Satzes an Bre.mer, Thim, Ter.min, Höschen, Rösch, drei oder vier Dipl.-Ings und an E. Kereszturi. Der, wichtig, “Dr. Med” ist.

    ist aber eben das genuine Wesen der axiomatischen Methode, dass ein Axiom nicht “abgeleitet” werden kann. ES BESTEHT ABER IMMER DIE MÖGLICHKEIT DAS ZU WIEDERLEGEN

    Ein Axiom widerlegen? Auftritt Wikipedia:
    Axiom im Sinne…

    • eines evidenten Grundsatzes (klassischer Axiombegriff):
      Der klassische Axiombegriff wird auf Euklid und Aristoteles zurückgeführt. „Axiom“ bezeichnet klassisch ein unmittelbar einleuchtendes Prinzip.[…] Als evidentes Prinzip bedarf ein Axiom weder eines Beweises, noch ist es einem Beweis zugänglich.

    [*hüstel*… klingt nicht sehr widerlegbar…]

    • eines allgemeinen Naturgesetzes (naturwissenschaftlicher Axiombegriff)
      In den empirischen Wissenschaften bezeichnet man als Axiome auch grundlegende Gesetze, die vielfach empirisch bestätigt worden sind. Als Beispiel werden die Newtonschen Axiome der Mechanik genannt.
      Auch wissenschaftliche Theorien, insbesondere die Physik, beruhen auf Axiomen. Aus diesen werden Theorien geschlussfolgert, deren Theoreme und Korollare Vorhersagen über den Ausgang von Experimenten treffen. Stehen Aussagen der Theorie im Widerspruch zur experimentellen Beobachtung, werden die Axiome angepasst. Beispielsweise liefern die Newtonschen Axiome nur für „langsame“ und „große“ Systeme gute Vorhersagen und sind durch die Axiome der Speziellen Relativitätstheorie und der Quantenmechanik abgelöst bzw. ergänzt worden.

    [Auch hier würd ich demzufolge eher die aus den Axiomen abgeleiteten Theorien widerlegen als die Axiome selbst. Wie erinnern uns: Axiome = Grundsatzannahme. Annehmen kann ich aber, was ich will.]

    • eines unabgeleiteten Ausgangssatzes einer Theorie (moderner (formaler) Axiombegriff)
      Durch Hilbert (1899) wurde ein formaler Axiombegriff herrschend: Ein Axiom ist jede unabgeleitete Aussage. Dies ist eine rein formale Eigenschaft. Die Evidenz oder der ontologische Status eines Axioms spielt keine Rolle und bleibt einer gesondert zu betrachtenden Interpretation überlassen.
      Ein Axiom ist dann eine grundlegende Aussage, die
      * Bestandteil eines formalisierten Systems von Sätzen ist,
      * ohne Beweis angenommen wird und
      * aus der zusammen mit anderen Axiomen alle Sätze (Theoreme) des Systems logisch abgeleitet werden.

    [Däääämn. Schon wieder “ohne Beweis” – also auch “ohne Gegenbeweis”.]
    Tja … wenn ich irgendwo einen Denkfehler gemacht hab, dann raus damit. Bis dahin allerdings gehe ich von einem lauwarmen Lüftchen aus, das aus des Endres Richtung weht.

  97. #97 Adent
    27. Oktober 2011

    @Bullet
    Naja, wenn da schon steht AXIOMA PHYSICA HUNGARICA dann ist dieses Axiom natürlich nur lokal in Ungarn evident, was Herrn Kereszturis Verwirrung auch einigermaßen erklären würde.

  98. #98 Florian Freistetter
    27. Oktober 2011

    @kereszturi: “Es ist ganz klar, dass dieser “Widerstand” von Seiten hoch(?)intelligenter Leuten nur tiefenpsychologisch zu deuten ist.”

    Na selbstverständlich! Was denn auch sonst? Wenn jemand sie kritisiert, dann kann das doch nur und ausschließlich aus irgendwelchen kranken, psychologischen Gründen geschehen. Und keinesfalls, weil ihre “Theorien” Unsinn ist. Das kann man absolut ausschließen, nicht wahr?

    “Lieber Herr Kollege, diesen Fall werden noch auch andere “Begutachter” bewerten – und darin auch IHRE “FACHMEINUNG”! Ich sorge dafür! “

    Na dann los! Aufschreiben, einreichen und wenn der Artikel dann veröffentlicht wird verspreche ich ihnen, dass ich ganz ausführlich darüber berichten werde. Aber sie werden sich lieber weiter den Internetrevoluzzer und das verkannte Genie spielen wollen. Ist auch eine wesentliche dankbarerer Rolle als der Realität ins Gesicht blicken zu müssen und auf die Fachgutachter zu warten, die ihnen erklären dass ihre pompöse Axiomatik Unsinn ist…

    “es ist zum Teil tragisch, zum Teil lächerlich, wenn man Leute begegnet, die behaupten, dass sie schon die “volle Wahrheit” besitzen… “

    Ja, das denke ich mir auch jedesmal wenn ich Typen wie sie treffe, die lieber lang und breit über tiefenpsychologische Gründe der Ablehnung ihrer “Theorie” schwafeln anstatt die Möglichkeit in Betracht zu ziehen, sie könnten sich geirrt haben. Echte Wissenschaftler dagegen wären höchst begeistert, könnten sie eine wissenschaftliche Revolution mit erleben oder ein paar neue Naturgesetze ableiten. Keiner würde behauptet, man wüsste schon alles.

    “Nun habe ich alles gesagt, was ich momentan zu sagen habe “

    Ach, könnte man das nur glauben…

  99. #99 Wurgl
    27. Oktober 2011

    Gerade die von mir angegebenen Werte von der Gravitationskonstante und von der GESAMTMASSE des Sonnensystems werden durch neuesten hochpräzisen Messungen bestätigt.

    Also wenn das nicht Radosophie ist, dann fress ich einen Besen (samt Putzfrau und den Eimer mit Putzwasser zum Nachtisch).

    Das ist ein pur heliozentrisches Weltbild. In diesem Satz ist zusätzlich noch die Aussage versteckt, dass in anderen Planetensystemen die Gravitationskonstante einen anderen Wert hat. Das ist Blödsinn pur, dazu braucht man keinen akademischen Titel, dazu braucht man nicht mal viel graue Masse zwischen den Ohren — das einzige was dazu notwendig ist, ist ein kurzes Nachdenken.

  100. #100 noch'n Flo
    27. Oktober 2011

    Sein Name ist Hase – er weiss von nichts…

  101. #101 Adent
    27. Oktober 2011

    @Endre Kereszturi
    Ach, btw, sie dürfen mich auch gern Igel nennen wenn ihnen mein Name Probleme macht.

  102. #102 ZetaOri
    27. Oktober 2011

    @endre kereszturi
    Sehr geehrter Herr Dr. Kereszturi, bezüglich Ihrer Aussagen …

    endre kereszturi· 26.10.11 · 16:28 Uhr
    … eines Tages werden meine Resultaten selbstverständlich auch bei der zuständigen Koryphäer der Physikwissenschaften vorgelegt.

    und …

    endre kereszturi· 27.10.11 · 07:50 Uhr
    … diesen Fall werden noch auch andere “Begutachter” bewerten – und darin auch IHRE “FACHMEINUNG”! Ich sorge dafür!

    … wäre es doch sicher nicht uninteressant, was Ihnen Horst Kremers von CODATA Germany auf Ihre “Information” vom 24.06.2008 (also vor mehr als 3 Jahren), sowie Prof. em. Eric Adelberger auf Ihr Schreiben vom 04.01.2009 (vor immerhin fast 3 Jahren) geantwortet haben. Würden Sie uns daran freundlicherweise teilnehmen lassen?

    @Adent· 27.10.11 · 08:36 Uhr

    Naja, wenn da schon steht AXIOMA PHYSICA HUNGARICA dann ist dieses Axiom natürlich nur lokal in Ungarn evident, …

    Tja, müsste man dann aber nicht statt der “Gesamtmasse des Sonnensystems” die Gesamtmasse Ungarns einsetzen (dürfte als Pyramide bis zum Erdmittelpunkt so etwa bei 10^21 kg liegen)? Das hätte natürlich einige Auswirkungen, z.B. auf den Wert der Gravitationskonstanten, was die Position des Herrn Dr. K. (mit beiden Beinen fest in der Luft) erklären könnte. >;->

    @ all
    Interessant auch (z.B. im Zusammenhang mit der Frage nach Vereinbarkeit von Wissenschaft und Religion im Parallel-Thread) sein Schlußsatz in der Kurzfassung seines “Werkes”:

    Die Hoffnung aber bleibt, dass sich Gottes Souveränität auch weiterhin im physikalischen Universum Ausdruck verschafft und in ihr der rechte Platz für die vorliegende Theorie gefunden wird.

    Hmm, hat er noch nicht gemerkt, dass sie diesen Platz schon hat?

  103. #103 endre kereszturi
    27. Oktober 2011

    Mein Kollege sollte einfach diese Zeilen überspringen – aber ich habe versprochen, auf sachliche Fragen offen zu antworten – damit schulde ich für meine Theorie…

    1./ Axiomatische Methode: In der SRT ist (Länge x Zeit) Lorentz-invariant. Wenn jemand damit “Radosophie” betreiben kann, kann es ohne weiteres tun: jede Beobachter wird diesen Multiplikationswert überall im Universum gleich finden – nur den Quotient [Längeeinheit(hier) x Zeiteinheit(hier)]/[Längeeinheit(dort) x Zeiteinheit(dort)] muss man dabei noch berücksichtigen. Insofern steht die metrologische Grundlage meines Axioms – (1 Meter x 1 Sekunde) – “jenseits von Gut und Böse”, also über allen theoretischen Zweifeln – auch “wikipedisch”… (:-) Dieser Wert ist EXACT 1 metsec. (ms wäre nicht eindeutig wegen Millisekunde.) Dieser Kern von AXIOMA PHYSICA HUNGARICA ist principiell nicht zu “wiederlegen”.

    Nun besteht aber mein Axiom noch aus zwei weiteren Teilen- diese sind selbstverständlich wiederlegbar. Es genügt nur zu beweisen, dass die vorgeführten Kombinationen von grundsätzlichen Naturkonstanten nicht den “notwendigen Werte” besitzen – HIER MUSS MAN ABER SCHON AUCH DIE MESSUNGENAUIGKEITEN IN BETRACHT ZIEHEN! Aber theoretisch sind diese Werte einfach nicht alle exact – nur c und mü_0 (vertreten in der Sommerfeldschen Feinstrukturkonstante). DARUM sagte ich, dass (diese Teile) wiederlegbar sind. Aber wenn jemand die CODATA-Werten betrachtet, könnte ruhig mein axiom auch heuristisch verwenden. Wenn doch einmal hier etwas “verbessert” werden muss, dann etwa so, wie einmal ein Eukleides-postulat (ursprünglich als “Axiom” betrachtet!) durch die Bolyai-Lobacsewsky- und Riemann- Geometrie wiedergelegt wurde.

    2./ Die Rolle der “Gesamtmasse des Sonnensystems”: Es ist in der Tat ungewöhnlich, dass dieser Wert so UNMITTELBAR mit mikrophysikalischen Konstanten korelliert. Aber bedenken wir einmal, dass alle Planeten – und auch die Sonne! – ihre Keplerschen Bewegungen im Sonnensystem so betätigen, dass in einem Fokus ihrer Ellipsenbahnen der MASSENMITTELPUNKT DES GESAMTEN SONNENSYSTEMS – und nicht einfach der Massenmittelpunkt der Sonne (WIE DAS GEWÖHNLICH (UND OBERFLÄCHLICH) ANGENOMMEN WIRD) – STEHT! Überlegt man einmal DIESE TATSACHE, dann wird auch eine bis jetzt nicht gewürdigte andere Tatsache verständlich: Nämlich das, warum die Lichablenkungswerte rund herum der Sonne bei einer Sonnenfinsternis NICHT GANZ SONNENCENTRISCH SYMMETRISCH SIND! Die Struktur der Raumzeit mit ihrer Geodätenbahnen – für Photonen unausweichlich zwingend – richtet sich nämlich nach dem Massenmittelpunkt des gesamten Sonnensystems – und dieser Punkt im Raum fällt mit dem Massenmittelpunkt der Sonne NICHT zusammen. Also muss man bei Auswertung dieser Aufnahmen auch “Astrologie” betreiben, und der aktuellen heliocentrischen Stand der Planeten berücksichtigen! Tun Sie das?!

    Nun kehre ich jetzt zu meiner Forschungen zurück – und auch mein Kollege kann weiterhin ruhig schlafen – sein “Weltbild” wird nicht mehr gestört – und es gibt noch viele leere Graben mit Patienten zu füllen… Mindestens bei dieser “Tätigkeit” haben wir gemeinsame Aufgaben… Nun Kopf hoch, und Toi, Toi, Toi! Grüsse: E.K.

  104. #104 noch'n Flo
    27. Oktober 2011

    Hatte sich der Magyare nicht schon längst verabschiedet? So ungefähr 20.000 Mal?

  105. #105 Florian Freistetter
    27. Oktober 2011

    @kereszturi: Sie wollten doch nichts mehr sagen. Schade

    “Dieser Kern von AXIOMA PHYSICA HUNGARICA ist principiell nicht zu “wiederlegen”. “

    Und dann wundern sie sich, dass sie niemand ernst nimmt…

  106. #106 s.s.t.
    27. Oktober 2011

    Irgendwie vermisse ich gearde den “gravitativen Hintergrund” und die “Oszillationen der Vakuolen”.

  107. #107 ZetaOri
    27. Oktober 2011

    Hallo Herr Dr. Kereszturi,

    wollen Sie uns wirklich NICHT berichten, welche “FACHMEINUNG” die “Koryphäer der Physikwissenschaften”, denen Sie Ihre Ergebnisse schon vor 3 Jahren (+/-) mitteilten, Ihnen gegenüber vertreten haben?
    Zumindest Prof. Adelberger sollte doch wohl derart REVOLUTIONÄRE Ergebnisse nicht einfach IGNORIERT haben? Immerhin hat er ja selber speziell zur Gravitationskonstanten GEFORSCHT.
    MfG
    Zeta

  108. #108 Wurgl
    27. Oktober 2011

    @endre kereszturi

    2./ Die Rolle der “Gesamtmasse des Sonnensystems”: Es ist in der Tat ungewöhnlich, dass dieser Wert so UNMITTELBAR mit mikrophysikalischen Konstanten korelliert.

    Das korrelliert in ihrem Kopf und sonst nirgendwo. Und genau das ist Radosophie.

    Wir Menschen leben zufällig in diesem Planetensystem und zufällig haben sie eine Formel gefunden die zu den von Ihnen gewählten Zahlen passt.

    Ihre Behauptung dass die Gravitationskonstante von der Sonnenmasse abhängt wird schon dadurch zu einer Büttenrede, da nur ein kleiner Schritt Verallgemeinerung dazu führt, dass ein jeder Zentralkörper der von anderen Körpern umkreist wird die jeweilige Gravitationskonstante bestimmt. Das ist schlicht und einfach absurd.

    Die andere Vorstellung, dass genau unsere Sonne die Gravitationskonstante für das ganze Universum festlegt ist ebenso hirnrissig. Was war denn vor der Entstehung der Sonne?

    Sorry, Sie haben sicherlich viel Lebenszeit geopfert um diese überaus tolle Formel zu finden, aber es war verschwendete Lebenszeit. Sehen und hören Sie sich doch einfach das Video von Harald Lesch an, es wird dort auch ein wenig Mathematik angesprochen und auch die Anzahl der möglichen Formeln genannt, die man aus wenigen Konstanten und ein paar Funktionen die man auf Zwischenergebnisse anwendet bekommt.

    Lauschen sie besonders aufmerksam beim Teil ab Minute 8. Wenn sie noch einen Rest an Kritikfähigkeit haben, dann werden sie ihren Fehler erkennen.

    PS: Video nochmals verlinkt, BR Alpha hat offenbar die Seite umstrukturiert.

  109. #109 Brockhoff
    12. Februar 2012

    Hallo !
    Wenn die Expansionsgeschwindigkeit so groß wird, muss man dann relativistisch rechnen ?
    z.B. bei einer Rotverschiebung von z = 8
    Wie wird dann die Distanz d ermittelt ?
    Kann die Expansionsgeschwindigkeit überhaupt über die Lichtgeschwindigkeit gehen ?
    Danke
    HB

  110. #110 Brockhoff
    12. Februar 2012

    Hallo !
    Kann die Expansionsgeschw. größer als die Lichtgeschwindigkeit werden, oder muss
    dann relativischtisch gerec honet werden ?
    Wie ermittelt man die Distanz ?

    Danke
    HB

  111. #111 Alderamin
    12. Februar 2012

    Brockhoff·
    12.02.12 · 17:19 Uhr

    Hallo !
    Kann die Expansionsgeschw. größer als die Lichtgeschwindigkeit werden, oder muss
    dann relativischtisch gerec honet werden ?

    Die Expansionsgeschwindigkeit kann größer als die Lichtgeschwindigkeit werden. Das ist ein bisschen kompliziert, Martin Bäker hat einen =>guten Artikel dazu geschrieben.

    Wie ermittelt man die Distanz ?

    So: https://de.wikipedia.org/wiki/Entfernungsma%C3%9F

    Wie man sieht, geht da ganz massiv der kosmische Dichteparameter Ω (in seinen Bestandteilen für die dunkle Energie ΩΛ und für die Materie Ω0) und der Hubble-Parameter H0 mit in die Gleichungen ein, d.h. die Entfernung hängt davon ab, wie sich das Weltall mit der Zeit ausgedehnt hat.

    Die verschiedenen Entfernungsmaße bedeuten grob:

    Laufzeitentfernung: welchen Weg hat das Licht bis zu uns zurückgelegt? (Während der Laufzeit hat sich die Galaxie weiter von uns entfernt)

    Mitbewegte Entfernung: wo wäre die Galaxie jetzt, wenn man das Weltall anhalten könnte?

    Winkeldurchmesserentfernung: Das Weltall war früher kleiner, aber wenn wir in verschiedene Himmelsrichtungen schauen, sehen wir dieses frühere, kleinere Weltall über den gesamten Himmel gespreizt, deswegen erscheint es vergößert und die Objekte erscheinen aufgrund dieser Vergößerung näher zu sein. Die Winkeldurchmesserenfernung gibt diese verkleinert erscheinende Entfernung an.

    Leuchtkraftentfernung:
    Es gibt eine Entfernungsformel, bei der man aus der scheinbaren Helligkeit eines Objekts (so hell, wie das Objekt am Himmel erscheint) und seiner bekannten absoluten Helligkeit (die Helligkeit in einer gedachten Entfernung von 10 parscec = 32,6 Lichtjahren) seine Entfernung berechnen kann. Durch Rotverschiebung und Zeitdilation der Photonen erscheint das Licht ferner Galaxien schwächer, d.h. sie erscheinen nach dieser Formel weiter weg zu sein. Die Leuchtkraftentfernung gibt diese vergrößerte Entfernung an.

    Hier gibt es einen =>Rechner, mit dem man die Entfernungen für verschiedene z ausrechnen lassen kann.

  112. #112 Brockhoff
    13. Februar 2012

    Danke !
    Die Hubble-Glg. zXc = Ho x d soll nur für z<=0,14 funktionieren. Kann diese Gleichung auch für größere z benutzt werden, oder muss sie dann modifiziert werden und wenn ja wie ? HB

  113. #113 Brockhoff
    13. Februar 2012

    Danke !
    Die Hubble-Glg. zXc = Ho x d soll nur für z <= 0,14 funktionieren. Wie kann die Gleichung modifiziert werden, damit sie auch für z.B. z=8 benutzt werden kann. Wer kann mir einmal eine Lösung für den Abstand d mit z=8 zeigen ? HB

  114. #114 Alderamin
    13. Februar 2012

    @Brockhoff

    Die Hubble-Gleichung gilt nur, solange der heutige Hubble-Parameter H0 die Ausdehnungsgeschwindigkeit des Weltalls beschreibt. Das ist bei z=8 aber nicht mehr der Fall, da wird die Formel kompliziert, weil die Änderung des Hubble-Parameters einen komplizierten Verlauf hat.

    Was Du suchst, ist wahlweise die Comoving Distance oder die Light Travel Distance, die im =>englischen Wikipedia-Artikel in einfacherer Form wiedergegeben sind, als im deutschen. Ich weiß nicht, ob sich die Integrale dc(z) und dT(z) in geschlossener Form lösen lassen; vermutlich nicht, sonst stünde die Lösung da. Also musst Du numerisch integrieren. z’ ist dabei nicht die Ableitung von z, sondern einfach die Integrationsvariable; man hätte vielleicht besser x und demgemäß E(x) verwendet.

    Oder weiß es jemand besser? Niels?

  115. #115 Alderamin
    13. Februar 2012

    Oder einfach den oben verlinkten Rechner anschmeißen, dann kommt für z=8 eine Lichtlaufzeit von 13,014 Milliarden Jahren und eine Mitbewegte Entfernung von 29,824 Milliarden Lichtjahren heraus.

  116. #116 Brockhoff
    15. Februar 2012

    Hallo !
    Wie kann man sich das Universum vorstellen ? Ist es eine abgeflachte Kugel, die größer wird ? Wenn ich alle sichtbaren Galaxien betrachte, so könnte man doch an eine Kugelform glauben.
    hb

  117. #117 Florian Freistetter
    15. Februar 2012

    @Brockhoff: “Hallo ! Wie kann man sich das Universum vorstellen ?”

    Am besten gar nicht 😉 Aber ernsthaft: Das Universum ist mindestens 4-dimensional und ein 4D-Objekt können wir uns nicht vorstellen. Wie man sich trotzdem etwas über die Form des Universums denken kann, hab ich hier beschrieben: https://www.scienceblogs.de/astrodicticum-simplex/2010/05/symmetrien-und-die-form-des-universums.php

  118. #118 Brockhoff
    16. Februar 2012

    Die Hubble-Gleichung soll nur für z-Werte bis ca. 0,14 funktionieren. Warum aber wird es so kompliziert wenn die Rotverschiebung z groß wird ?
    hb

  119. #119 endre kereszturi
    21. Februar 2012

    Liebe, liebe Herren Professoren!

    Wie wir wissen, es kommt in der ART ein konstanter Wert in der Gleichung auch dort vor, wo die (relativistisch bedingte Zusatz-)Periheldrehung der Planetenbahnen im Sonnensystem angegeben wird: 6Pi x G x M/c^2. (G die Gravitationskonstante, M die Sonnenmasse, c die Vakuumlichtgeschwindigkeit.) Dieser konstanter Wert – etwa 27834 Meter – geteilt durch die Planetenbahngrossachse/2 bei Berücksichtigung der Bahnexzentrizität ergibt exact den Winkelmass der Periheldrehung, welche zusätzlich zur anderen Planetenstörungen rein ART-mässig hinzukommt.

    Nun fand ich heraus, dass die Mondbewegungen – vermittels natürlich der Planet Erde – ebenso von diesem konstanten Wert abhängig sind. Und zwar so: der synodische Mondmonat dauert 29,530589 Tage (1 d – astronomisch – exact 86400 s), der siderische 27,321661 Tage. — 29,530589^4 geteilt durch 27,321661 = 27834,366…
    Berücksichtigt man auch die Dimensionen – ist doch ein Pflicht, oder! – kommt man auf einen “ergänzenden” Ausdruck von (1 Tag)^3/1 m. Unser Team grübelte lange darüber, was das dann eigentlich sein könnte?… Der beste Vorschlag lautete: stellen wir um: (1 Tag)^3/1 m = (86400 s)^3/1 m === (1 s)^3/1,550453596…e-15 Meter! DIESE Längekonstante hatte schon Heisenberg in seinem “Weltformel” herausgearbeitet – aber unabhängig davon ist dieser Wert auch in dem Standard Modell gut bekannt – ältere Beispiele (wie etwa der “klassische Elektronradius” etc.) lasse ich bei Seite…

    Dass diese Lösung des “relativistischen Dreikörperproblems” (ich meine Sonne – Erde – Mond) mit Hilfe der erwähnten ART-Konstante unmittelbar zu einer mikrophysikalischen “Längekonstante” führt, lässt mich hoffen, dass meine “Einheitliche Theorie der Naturkonstanten” doch etwas mehr ist, als das bei Ihnen “vermutet” wird. — Radosophie ? (:-) Numerologie ? (:-) Aber meine Herren, machen Sie sich vor der zukünftigen Wissenschaftler-Generationen doch nicht lächerlich – mein Archiv bewahrt Ihre Reaktionen… Ansonsten herzliche Grüsse: E.K.

  120. #120 Thomas J
    21. Februar 2012

    @endre

    naaaa, wieder Ferien und Zeit der Radosophie zu frönen? Freut mich für Sie, schön, wenn man so viel Zeit hat in seinem Leben 🙂

  121. #121 Florian Freistetter
    21. Februar 2012

    @endre: “Wie wir wissen, es kommt in der ART ein konstanter Wert in der Gleichung”

    Sind sie nicht gesperrt? Bitte verbreiten sie ihre Privatphysik anderswo, aber nicht hier. Danke.

  122. #122 stillerleser
    21. Februar 2012

    Den gibts ja immer noch! 🙂

    Herr Kereszturi, wie finden Sie neben all ihrer “Forschung” und Privatkorrespondenz mit den weltweit führenden Professoren nur die Zeit diesen Blog der ahnungslosen Laien mit ihren revolutionären Ergebnissen zu bereichern? Welch ungeheure Ehre, dass Sie ihre phantastischen Erkenntnisse nicht auf internationalen Tagungen und in Fachzeitschriften veröffentlichen sondern über diesen kleinen Astronomieblog der Welt mitteilen.
    Vielen Dank.

  123. #123 Oliver Debus
    21. Februar 2012

    Liebe, liebe Herren Professoren!

    hmmm, war mir doch schon länger so, dass ich was vergessen habe. Was war das nur??

    Hach, jetzt habe ich es wieder, vielen Dank Herr Kereszturi. Ja, habe doch glatt vergessen, dass ich vor langer Zeit meine Habitilation gemacht habe. Sie haben mich gerade an meinen Professorentitel erinnert.

    Also für alle, hier schreibt in Zukunft Prof. Dr. Dr. multi h.c. O. D. 😀

    Den Rest ihres Kommentars kommentier ich lieber nicht.

  124. #124 Alderamin
    21. Februar 2012

    Nachtrag an

    @Brockhoff

    Die Hubble-Gleichung soll nur für z-Werte bis ca. 0,14 funktionieren. Warum aber wird es so kompliziert wenn die Rotverschiebung z groß wird ?

    Weil für größere Werte der Hubble-Parameter keine Konstante mehr ist, sondern in einem Wechselspiel von Gravitation zwischen den Galaxien und der dunklen Energie erst ab- und dann wieder zugenommen hat. Für zunehmende z sieht man sozusagen diese gesamte Entwicklung vor sich und muss sie in der Entfernugnsformel mit berücksichtigen.

  125. #125 Hans
    22. Februar 2012

    @Adent: Kaufen, als Geschenk an Reich-Ranicki senden, Buchrezension abwarten. Nicht den Tag verderben lassen. Das ist so ein Buch nicht wert!

  126. #126 Adent
    22. Februar 2012

    @Hans
    1. Viel zu teuer (Hardcover)
    2. Ich mag Reich-Ranicki, würde ihm ungern einen Herzinfarkt verpassen wollen ;-).

  127. #127 Brockhoff
    25. Februar 2012

    Hallo !
    Auf die Gefahr hin, das ich nerve, habe ich die Frage :
    Wie berechnet man den Abstand, falls z=8,0 ist ?
    Inwieweit geht die SRT dort ein ?
    Wie groß ist der Fehler ?
    Danke
    hb

  128. #128 Bjoern
    25. Februar 2012

    @Brockhoff: Zunächst mal ist bei so großen Rotverschiebungen sowieso erst mal fraglich, was man unter “Abstand” überhaupt genau verstehen soll… Florian hat vor ein paar Wochen dazu mal was geschrieben:
    https://www.scienceblogs.de/astrodicticum-simplex/2012/01/die-rotverschiebung-und-die-vielen-entfernungen-der-kosmologie.php

    Die SRT ist da übrigens ziemlich irrelevant; wichtig ist die ART. Wie groß der Fehler ist, weiss ich leider auch nicht genau (müsste man wohl in der Veröffentlichung selbst nachschauen), aber ich würde schätzen, maximal 10% – die Verfahren sind inzwischen ziemlich ausgereift.

  129. #129 Alderamin
    25. Februar 2012

    @Brockhoff

    Hatte ich Dir oben schon beantwortet:

    Die augenblickliche Entfernung zu einem Objekt bei z=8, das wir jetzt gerade sehen, ist

    d = c/H0 * ∫(0->8) [dz / √(Ωm *(1+z)³ + (1+z)² + ΩΛ)]

    mit
    ∫(0->8)[…] Integral von 0 bis 8 (Dein gewünschtes z) über den Ausdruck in in den eckigen Klammern
    c = Lichtgeschwindigkeit = 300000 km/s
    H0 = heutiger Hubble-Parameter =71 km/s / Mpc (wenn Du die Entfernung in Mparsec haben willst; 1 Mpc = 32,6 Millionen Lichtjahre)
    Ωm = Dichte der sichtbaren und dunklen Materie = 0,27
    ΩΛ = Dichte der dunklen Energie = 0,73

    Das Integral konvergiert für kleine z gegen z und Du bekommst die Hubble-Formel raus. Für große z kann man es wohl nicht einfach nach z auflösen, sonst gäbe es eine einfachere Formel, das musst Du numerisch lösen, kannst Dir ein Programm schreiben, Matlab, Mathematica oder ähnliche Programme benutzen, oder einfach =>den kosmologischen Rechner benutzen. Da kommt dann unter comoving radial distance der gewünschte Wert raus. Genau so und nicht anders rechnet man das. Tut mir leid, dass es nicht einfacher geht.

    Quelle: https://en.wikipedia.org/wiki/Distance_measures_%28cosmology%29

  130. #130 Alderamin
    25. Februar 2012

    @myself

    Korrektur: Der Faktor für den zweiten Term in der Wurzel (die Krümmung des Raums) ist 0, nicht 1, nur dann konvergiert das Integral gegen z für kleine z. Es muss also richtig heißen:

    d = c/H0 * ∫(0->8) [dz / √(Ωm *(1+z)³ + ΩΛ)]

    Das macht das Integral einfacher, aber es scheint trotzdem nicht geschlossen lösbar zu sein, ich habe in den Tabellen im Bronnstein-Semendjajev jedenfalls keine passende Stammfunktion gefunden.

    Hat jemand ein Mathematik-Programm und kann das mal nachprüfen?

  131. #131 Alderamin
    25. Februar 2012

    nochmal @myself

    Ich hab’ mir gerade das kostenlose =>Maxima geladen (geiles Tool), und das kann das Integral jedenfalls nicht geschlossen lösen.

    Es kann es aber numerisch lösen:
    Das Integral alleine hat von 0 bis 8 den Wert 2,166507804543789. In die Formel eingesetzt mit H0=71 km/s/Mpc und c=299792,5 km/s kommt raus: 9147,9266 Mpc. Mal 3,26 Millionen Lichtjahre sind das 29,8222 Milliarden Lichtjahre.

    Der Cosmology Calculator liefert 9144,0 Mpc oder 29,824 Milliarden Lichtjahre. Ist doch ganz brauchbar. Wo jetzt der Fehler in der 5. signifikanten Stelle kommt, weiß ich nicht, jedenfalls ist der Fehler durch H0 (70,8 +/- 1,6 km/s) schon wesentlich größer.

  132. #132 Brockhoff
    29. Februar 2012

    Hallo !
    Muss die Gleichung nicht d = c/H0 * ∫(0->8) [dz / √(Ωm *(1+z)³ + 1 + ΩΛ)]
    sein, denn (1+0)^2 = 1 .
    Der Wert ändert sich von 2,1…… auf 1,9……

    hb

  133. #133 Brockhoff
    29. Februar 2012

    Entschuldigung !
    Mein Argument über die Krümmung des Raums (1+0)^2 = 1 ist Blödsinn !

    hb

  134. #134 Brockhoff
    8. März 2012

    Hallo !
    Wenn ich die Formel d = c/H0 * ∫(0->8) [dz / √(Ωm *(1+z)³ + 1 + ΩΛ)] zugrunde lege,
    wie groß wäre dann die Geschwindigkeit bzw. die Zeit ?

    Danke

  135. #135 Alderamin
    8. März 2012

    @Brockhoff

    Wenn Du die Fluchtgeschwindigkeit meinst:

    1+z = √[(1+v/c)/(1-v/c)] nach v auflösen (siehe =>Kasten). Du hattest z=8 vorgegeben.

    Was meinst Du mit Zeit? Lichtlaufzeit, Zeitdilatation, oder was anderes? Zeitdilatation ist jedenfals 1+z. Alles läuft mit diesem Faktor verlangsamt ab, z.B. dauern Supernovae bei z=8 neunmal so lange wie bei uns in der Nähe.

  136. #136 Brockhoff
    8. März 2012

    Danke für die Antwort !
    Wenn ich das Licht eines Objekt analysiere, und ich stelle eine Rotverschiebung z fest,
    so kann mit Hilfe der Beziehung d = c/H0 * ∫(0->8) [dz / √(Ωm *(1+z)³ + 1 + ΩΛ)] den Abstand vom Beobachter feststellen. Da ich relativistisch rechnen muss, kann ich die Geschwindigkeit nach 1+z = √[(1+v/c)/(1-v/c)] ermitteln. Wenn ich davon ausgehe, dass sich das Universum nicht beschleunigt ausdehnt, so könnte ich über d und v die Zeit ausrechnen, die das Objekt für die Entfernung benötigt hat. Oder ?
    Bei allen Objekten sollte doch die gleiche Zeit ( 13,7 Milliarden Jahre ) herauskommen,
    oder ?
    HB

  137. #137 Alderamin
    8. März 2012

    @Brockhoff

    Wenn ich davon ausgehe, dass sich das Universum nicht beschleunigt ausdehnt, so könnte ich über d und v die Zeit ausrechnen, die das Objekt für die Entfernung benötigt hat. Oder ?

    Nicht so einfach. Das Objekt hat sich nicht bewegt, sondern der Raum dazwischen ist gewachsen. Und auch bei konstanter Ausdehungsgeschwindigkeit wächst die Strecke nicht linear. Nehmen wir an, die Strecke x wächst um den Anteil deltax pro Zeiteinheit. Wenn x sich auf 2x vergrößert hat, dann wächst 2x pro Zeiteinheit mit 2*deltax. Das ist ein exponentielles Wachstum. Du hast also immer eine wachsende Zunahme der “Geschwindigkeit” (besser: der Rotverschiebung). Du kannst nicht einfach mit konstanter Rotverschiebung rechnen, t=d/v oder so.

    Bei allen Objekten sollte doch die gleiche Zeit ( 13,7 Milliarden Jahre ) herauskommen,
    oder ?

    Das schon mal gar nicht, wenn Du die dunkle Energie vernachlässigst, dann kommt weniger heraus. Bevor die dunkle Energie entdeckt war, ging man noch von einem Weltalter von ca. 11 Milliarden Jahren aus. Was weniger war als das Alter der ältesten Sterne war (hierbei wurde allerdings eine Abbremsung der Ausdehnung angenommen). Wenn Du von einem konstanten Hubble-Parameter ausgehst, kommt irgendwas heraus. Möglicherweise bei allen Objekten dasselbe “irgendwas”, bin da aber nicht ganz sicher.

  138. #138 Brockhoff
    9. März 2012

    Hallo !
    Ich habe folgende Lösung in der Literatur gefunden :
    z=7,1 , Zustand : 770 Mill. km nach dem Urknall, Licht = 12,9 Milliarden Jahre unterwegs

    Ich möchte diese Werte nachrechnen können. Wer kann helfen?
    Danke
    hb

  139. #139 Alderamin
    9. März 2012

    @Brockhoff

    Ist doch alles oben schon verlinkt.

    Du suchst die =>Light Travel Distance.

    Formel: dT(z) = c/H0 * ∫(0->z) [dx / ((1+x)*√(Ωm *(1+x)³ + ΩΛ))]

    Setze ein z=7,1, H0 = 70,8 km/s/Mpc, Ωm=0,27 , ΩΛ=0,73 und löse das Integral, z.B. mit Maxima. Das Ergebnis ist wegen der Wahl der Einheiten von H0 in Mpc, das muss man dann noch mit 32,6 Millionen Lichtjahren/MPc multiplizieren, und dann sollten Deine 12,9 Milliarden Lichtjahre herauskommen. 770 Millionen Jahre nach dem Urknall ergibt sich aus dem Weltalter von 13,75 Milliarden Jahren (da gibt’s eine Unsicherheit von +/- 0,13 Milliarden Jahren, daher sind die 770 Millionen nicht ganz wörtlich zu nehmen).

    Wozu willst Du denn das alles so genau wissen? Und warum nimmst Du nicht den oben schon mal verlinkten Cosmology Distance Calculator? Wenn Dir die Integrale und die Wiki-Artikel anscheinend zu kompliziert sind?

  140. #140 Alderamin
    9. März 2012

    Nachtrag:

    Für H0=71 km/s/Mpc kommt es im =>Cosmology Calculator noch besser hin als für 70,8. Einfach z=7,1 einsetzen und auf “Flat” klicken. “Light Travel Time” und “Age at Redshift z” liefern die von Dir gefundenen Werte.

  141. #141 Brockhoff
    13. März 2012

    Hallo !
    Wozu wiilst du das alles……
    Was sagt Einstein : Neugier ist wichtiger als Wissen !

    hb

  142. #142 Alderamin
    13. März 2012

    @Brockhoff

    Ich dachte, Du schreibst einen Sci-Fi-Roman oder so was 😉
    Nein, Neugier ist natürlich lobenswert.

    Ich hoffe, Du kannst jetzt halbwegs nachvollziehen, wie man auf die Zahlen kommt, ich hatte z=8 ja weiter oben schon mal mit Maxima nachgerechnet, und es passte recht gut. Mit der Herleitung der Formeln aus der Wikipedia kann ich allerdings nicht dienen, das ist mir dann auch zu hoch.

  143. #143 Thomas Grimberg
    1. Juli 2012

    Wie kann dass sein : Eine Galaxie in einer Entfernung von einem Megaparsec bewegt sich von uns also mit 73.8 Kilometern pro Sekunde (immerhin 265700 km/h) weg. Eine Galaxie in einer Entfernung von 2 Megaparsec ist doppelt so schnell: 147.6 Kilometer pro Sekunde – usw. ??

    Es heißt doch, das ab Lichtgeschwindigkeit alles Materie sich auflöst, so Brian Greene.

  144. #144 Florian Freistetter
    1. Juli 2012

    @Thomas Grimberg: Die Galaxie bewegt sich nicht durch den Raum. Der Raum selbst zwischen den Galaxien ist es, der sich ausdehnt. Ist also kein Problem für die Materie.

  145. #145 Thomas Grimberg
    1. Juli 2012

    ..das Weltall, also Materie dehnt sich aus, expansiv , währendessen sich der Raum auch ausbreitet ?

  146. #146 Thomas Grimberg
    1. Juli 2012

    Wie können wir uns das vorstellen, das sich die Galaxien, fast mit zweifacher Lichtgeschwindigkeit entfernen ? Heißt dass, das der Raum dazwischen Überlichtschnell ausdehnt ?

  147. #147 josef
    Neualbenreuth
    12. November 2012

    Die Tatsache, dass die Messungen ergeben, dass je weiter eine Galaxie oder Stern sich von uns befindet die Expansionsgeschwindigkeit größer wird, ist keineswegs ein Widerspruch zur älteren Annahme, dass durch die Gravitation eigentlich die Expansion langsamer werden sollte. Be der “neuen” Überlegungen wird nicht berücksichtigt, dass eben, je weiter ein Objekt von uns entfernt ist, desto weiter wir in die Vergangenheit gelangen, die höher genesene Geschwindigkeit, ist also die Geschwindigkeit der Vergangenheit, also umgekehrt als gedacht, in der Gegenwart verlangsamt sich die Expansionsgeschwindigkeit. Es erscheint mir unerklärlich, dass nahmhaften Wissenschaftler dies nicht berücksichtigen!

  148. #148 Phero
    12. November 2012

    Es würde helfen, wenn du deine Gedanken ein wenig klarer formulieren tätest.
    Du meinst also, da wir bei weiter entfernten Galaxien eine höhere Expansionsgeschwindigkeit messen, träfe dies eben nur für weit entfernte Galaxien zu und damit nur für “früher”. So einfach ist das aber nicht.
    Um einmal meine Kentnisse in Kosmologie herauszukramen: Die gemessene Rotverschiebung kommt vor allem dadurch zustande, dass sich der Raum ausdehnt und somit die Wellenlänge der Photonen streckt, mit einem Dopplereffekt hat das nichts zu tun. Das bedeutet aber, dass sich der Raum während der gesamten Zeit, die das Licht zurückgelegt hat, dementsprechend gestreckt hat.
    Ich bin mir zwar nicht sicher, dass dieses Argument deines widerlegt – aber wenn ich der Meinung wäre, “dass namhafte Wissenschaftler dies nicht berücksichtigen” wäre es vielleicht erst einmal besser, einfach nachzufragen – anstatt sofort mit einer definitiven Aussage aufzuschlagen.

  149. #149 Alderamin
    12. November 2012

    @josef

    Es erscheint mir unerklärlich, dass nahmhaften Wissenschaftler dies nicht berücksichtigen!

    Selbstverständlich wird das berücksichtigt, man kann sogar nachweisen, dass sich in den ersten 7 Milliarden Jahren die Expansion verlangsamte, dann aber die Dunkle Energie die Oberhand gewann und sich von da an die Expansion wieder beschleunigte. Die Kosmologen sind ja keine Deppen.

    https://www.astronomy.com/en/News-Observing/News/2003/10/Distant%20supernovae%20trace%20early%20universes%20deceleration.aspx

    https://www.eso.org/~bleibund/papers/EPN/epn.html

  150. #150 Alderamin
    12. November 2012

    @josef

    Noch ergänzend: wenn man in die Ferne blickt, sieht man nicht nur die Expansion des Universums zur betreffenden Zeit, in der das beobachtete Objekt sein Licht auf den Weg schickte, sondern man sieht den aufsummierten Expansionseffekt des Universums während der gesamten Reisezeit des Lichts, weil das Licht der Expansion des Universums auch unterworfen war, als es unterwegs war.

    Deswegen ist die Formel für die Laufzeitentfernung, also die Lichtlaufzeit zwischen zwei Objekten mit verschiedener Rotverschiebung z, auch so ein komplizierter Ausdruck, der über ein Integral berechnet wird. Es gehen dort sowohl die kosmische Materiedichte Ω0, die alleine für sich genommen zu einer Abbremsung der Expansion führen würde, als auch die Dichte der Dunklen Energie ΩΛ ein, die das Weltall auseinander treibt. ΩΛ ist derzeit fast dreimal so groß wie Ω0.

  151. #151 Bullet
    12. November 2012

    Es erscheint mir unerklärlich, dass nahmhaften Wissenschaftler dies nicht berücksichtigen!

    Dunning-Kruger in the wild…

  152. #152 noch'n Flo
    Schoggiland
    12. November 2012

    @ Bullet:

    Und natürlich wieder einmal:

    Form & Inhalt.

    (Wie konntest Du das nur vergessen?)

  153. #153 der kleine klingone
    niederkassel
    29. März 2013

    Ich habe gehört, dass sich das Universum pro sek. ungefähr um 200 Mio. Kilometer ausdehnt.

  154. #154 der kleine klingone
    niederkassel
    29. März 2013

    Expansionsverlauf des Universums, berechnet nach der Zeit/Radius-Konstanten des Universums
    Daten unseres Universums

    kt/R = 1,215314191… × 1043 m3 / s2 = G × ½ MU = Rumax × c2
    kt/R = 1,352219403… × 1026 m × 8,987551787… × 1016 m2 / s2

    Mittels der Radius-Zeit-Konstanten des Universums lässt sich bei gegebener Zeit in Sekunden nach dem Urknall der zu diesem Zeitpunkt erreichte Radius des expandierenden Universums berechnen.
    Wenn dagegen der Radius des Universums in Metern gegeben ist, dann läßt sich mit der Konstanten der Zeitpunkt in Sekunden nach dem Urknall berechnen.

  155. #155 der kleine klingone
    niederkassel
    29. März 2013

    r = 1,4 × 10 10 Lj (Durchmesser d = 28 Milliarden Lichtjahre)

  156. #156 Nils
    27. November 2013

    Hallo Florian,

    welches Riess paper meinst Du hier in dem die Daten zur Wiederlegung der dunklen Energie durch eine Void-Alternative belegt werden?

    Hast Du vllt die arXiv Nr?

    Vielen Dank im voraus.

  157. #157 Florian Freistetter
    27. November 2013

    @Nils: Was genau meinst du (den Artikel hier hab ich vor mehr als zwei Jahren geschrieben, da weiß ich nicht mehr jedes Detail). Geht es um den Artikel von Adam Riess von dem mein Artikel handelt? Den hab ich ja direkt im Text verlinkt: “Die Ergebnisse sind in einem Artikel mit dem Titel A 3% solution: Determination of the Hubble Constant with the Hubble Space Telescope and Wide Field Camera veröffentlicht wurden”…

  158. #158 Nils
    3. Dezember 2013

    Vielen Dank Florian,
    ich habe angenommen es gäbe noch einen weiteren Artikel zu dem Thema, der die Behauptung:
    “… Nähe der Mitte dieser Blase leben würden, dann könnte das den Eindruck der beschleunigten Expansion bewirken. Aber diese Hypothese konnte nun mit den neuen Daten von Riess et al. widerlegt werden.” stützt. Doch ich habe das Resultat des Artikels etwas anders aufgefasst, nämlich dass komplexere Voidmodell zur Nachahmung dunkler Energie nicht ausgeschlossen sind. Über die Existenz von dunkler Energie, bzw deren Anteil am Gesamten kann man durchaus weiterhin geteilter Meinung sein, wie ich finde. Alles in Allem ist Dir aber ein durchaus ansprechender Artikel gelungen.

  159. #159 nobotter
    24. Juli 2014

    Das Universum kann sich prinzipiell auch ohne “Dunkle Energie” oder “Dunkle Materie” beschleunigt ausdehen.

    E / (m*s) = a

    E=Energie (des Universums)
    m=Masse ( gemäß E=m*c*c aus Energie kondensierte Materie)
    s=Entfernung (vom Ausgangpunkt “Urknall”)
    a=Beschleunigung des Universums

    Bzw. für Raumsektoren x :
    Ex / (mx * sx) = ax

    Und (Summe Ex) = E und (Summe mx)=m
    E = Gesamtenergie des Universum
    m = Gesamtmasse des Universums
    ax = Beschleunigungswert in Raumsektor x

    Der “Urknall” findet immer noch statt. Das wurde hier auch in 2011 schon mal angedacht. Es ist nach der Formel zu erwarten, dass sich die Beschleunigungswerte in der Vergangenheit geändert haben und vlt. auch wieder ändern,
    wenn Materie aus Energie entsteht. Insgesamt geht a aber gegen 0, da s immer größer wird. Das ist auch zu erwarten, da mit zunehmender Expansionsgeschwindigkeit gegen die Lichtgeschwindigkeit die Kraft für eine weitere Beschleunigung der Massen gegen unendlich geht.

    Das materielle, massenbehaftete Universum sollte auch keine Kugel mehr sein, denn im Laufe der Zeit wird es unterschiedliche Beschleunigungswerte ax gegeben haben.

  160. #160 Harry Trick
    Berlin
    11. März 2015

    Hallo, Florian,
    bin durch Zufall auf diesen Blog gestoßen, den ich sehr informativ finde. Ein Problem beschäftigt mich aber doch sehr:
    Wenn sich das Universum ausdehnt, also sich alles von allem entfernt, wie soll es dann möglich sein, dass unsere Milchstraße in ein paar Millionen Jahren mit der Andromeda-Galaxis “zusammenstößt”?
    Danke und Gruß aus Berlin
    Harry Trick

  161. #161 Alderamin
    11. März 2015

    @Harry Trick

    Wenn sich das Universum ausdehnt, also sich alles von allem entfernt, wie soll es dann möglich sein, dass unsere Milchstraße in ein paar Millionen Jahren mit der Andromeda-Galaxis “zusammenstößt”?

    Mehr noch: wieso konnten überhaupt Galaxien und Galaxienhaufen aus ursprünglich gleichverteiltem Gas entstehen? Wie Sterne und Planeten? Warum wächst das Sonnensystem und wir alle nicht mit?

    Weil der Effekt der Raumexpansion äußerst winzig ist und nur über sehr große Entfernungen die Geschwindigkeiten erreicht, mit denen sich Galaxien relativ zueinander bewegen. Auf Entfernungen, bei denen die Schwerkraft eine größere Fluchtgeschwindigkeit erfordert, als die Raumexpansion sie aufbringt, dominiert die Schwerkraft und die Objekte bleiben aneinander gebunden. Deswegen wirkt die Schwerkraft über einige zehn Millionen Lichtjahre Entfernung stärker als die Raumexpansion. Und das gilt erst recht für Objekte, die durch die viel stärkeren elektromagnetischen Kräfte aneinander gebunden sind. Die rutschen gewissermaßen über den Raum hinweg, der unter ihnen expandiert. Deswegen wachsen die Sonne, die Erde und ihre Bewohner nicht mit.

    Nur deswegen konnten sich Strukturen im Universum überhaupt erst bilden, denn das ursprüngliche Gas konnte sich trotz der Raumexpansion verdichten und Filamente, Galaxienhaufen und Galaxien bilden. Innerhalb der Galaxienhaufen sind die Galaxien aneinander gebunden und vereinigen sich mit der Zeit zu größeren Galaxien. Genau das tun die große Andromedagalaxie und die Milchstraße ebenfalls in 5 Milliarden Jahren (und möglicherweise noch ein paar andere Galaxien in der lokalen Gruppe).

  162. #162 Florian Freistetter
    11. März 2015

    @Harry: “wie soll es dann möglich sein, dass unsere Milchstraße in ein paar Millionen Jahren mit der Andromeda-Galaxis “zusammenstößt”?”

    Es entfernt sich ja nicht “alles von allem”. Die Expansion wirkt sich nur auf sehr großen Abständen aus; bei kleineren Abständen ist die Gravitationskraft groß genug, um die Strukturen – wie zB Galaxien oder Galaxienhaufen – zusammenzuhalten. Aber ich denke, ich werde das demnächst mal ausführlich in der “Fragen zur Astronomie”-Rubrik behandeln (https://scienceblogs.de/astrodicticum-simplex/fragen-zur-astronomie/) – die Frage kommt nämlich sehr oft.

  163. #163 B. Kropp
    27. April 2015

    Vielleicht ein gewichtiger Einwand:
    Man schätzt allein die baryonische Masse in unserem
    Universum zu 10^53 kg. Bei einem Universumsradius von min. 10^14 mrd LY errechnet sich schon eine Dichte von z.B. 10^-25 kg/m³.
    Berechnet man die kritische Dichte eines SL für gleiche 10^53 kg erhält man Werte in der Größenordnung 10-95 kg/m³
    Ist damit bewiesen, dass unser Universum ein SL ist und kollabieren wird ?

  164. #164 Florian Freistetter
    27. April 2015

    @B. Kropp: “Ist damit bewiesen, dass unser Universum ein SL ist und kollabieren wird ?”

    Nein.

  165. #165 David R. Gollenia
    Bocholt
    17. Januar 2016

    Stimmt es, das bei der anfänglichen Expansion des Weltraums ein minimaler Geschwindigkeitsunterschied, von nur Trillionstel Sek. zu einem Verfall der Elemente geführt hätte und es somit keine Planeten, geschweige denn etwas lebendiges gegeben hätte?

  166. #166 Krypto
    17. Januar 2016

    @David:
    Unser Universum hat offenbar eine dermaßen präzise Feinabstimmung, sodass sich die Vermutung aufdrängt, dass es unzählige andere, “versagte” Universen geben könnte.
    Wir können aber absolut keine fundierte Aussage darüber machen, ob und wenn ja, in was für einem Glücksfall wir hier leben.
    Wir existieren hier deshalb, weil wir es können.
    Ob das völlig normal oder superselten für Durchschnittsuniversen ist, können wir jedoch nur vermuten.

  167. #167 Krypto
    17. Januar 2016

    Oops, vergessen:
    Google doch einfach mal nach “kosmologische Feinabstimmung”

  168. #168 David R. Gollenia
    46395 Bocholt
    18. Januar 2016

    @Krypto: Danke dir.

    Damit drängt sich mir mehr denn je nach einem intelligenten Verursacher auf, der ein Meister im Fach ist.

  169. #169 Folke Kelm
    18. Januar 2016

    David R. Gollenia,

    Das ist aber ein hohes Ross auf dem Du sitzt, auch wenn Du es vielleicht nicht selber so siehst, Aber denke doch mal nach, was Du sagst. Das Universum in dem wir leben, ist geschaffen nach unseren Vorgaben? Damit eben genau wir darin leben können?
    Diese kreationistische Sicht der Dinge ist in meinen Augen Hybris. Der Mensch als Mass aller Dinge, Gott oder Spaghettimonster erschafft das Universum nur für uns….
    Wie wäre es denn mal mit einer anderen Sichtweise, das Uiversum ist mit seiner “feinabstufung” eben genau jenes in dem wir uns entwikelt und angepasst haben, und es ist das einzige, was wir momentan sehen können.
    Mathematisch können sich unendlich viele UNiversen entwickeln, mit ihren eigenen feinabstufungen. IN jedem können sich vielleicht Intelligenzen entwickeln angepasst an genau diese Feinabstufung, und jede dieser Intelligenzen kann nur eben ihr eigenes Universum sehen.
    Wir sind in eben diesem perfekten Universum, weil wir uns in ihm und nach seinen Vorgaben entwickelt haben, nicht anders herum.
    Ausserdem ist das Universum gar nicht perfekt.

    David, ist es nicht so, dass Kreationisten sich selbst überheben, mit Hilfe ihres Werkzeuges “Gott”?

  170. #170 StefanL
    18. Januar 2016

    @168
    ID? Kaum.
    Weder “selten” noch “normal” impliziert die Notwendigkeit eines ID.

  171. #171 DR Gollenia
    Boh
    18. Januar 2016

    Nun ja. Es ist schon eine interessante Sache die Suche nach den Ursprung. Es wird wohl noch einiges an Zeit brauchen, eine befriedigende zuverlässige Antwort zu finden. Ich für meinen Teil kann einfach nicht nachvollziehen, wie so ein einzigartiges Ergebnis ohne einen intelligenten Designer entstanden sein soll. Daher, so lange mir keiner das Gegenteil beweisen kann, gehe ich davon aus, das eine Intelligenz, wie auch immer man sie nennen möchte, oder auch bezeichnet, unabdingbar ist.
    1 Thema 2 Meinungen
    MfG: Ich

  172. #172 Alderamin
    18. Januar 2016

    @DR Gollenia

    Ich für meinen Teil kann einfach nicht nachvollziehen, wie so ein einzigartiges Ergebnis ohne einen intelligenten Designer entstanden sein soll.

    Aber wie ein intelligenter Designer enststanden sein soll, wohlmöglich ganz ohne Universum und irgendwelche Naturgesetze, ist für Dich kein besonderes Problem? Da hapert’s bei meiner Vorstellungskraft, und zwar gewaltig.

    Daher, so lange mir keiner das Gegenteil beweisen kann, gehe ich davon aus, das eine Intelligenz, wie auch immer man sie nennen möchte, oder auch bezeichnet, unabdingbar ist.

    Solange Du mir nicht beweisen kannst, dass ich in der Garage keinen rosa Drachen habe, geh’ davon aus, dass es ihn gibt 😉

  173. #173 Bullet
    18. Januar 2016

    @DR Gollenia: ich finde es ehrlich gesagt nur begrenzt lustig, wie wenig du zuzuhören bereit bist, wenn das zu hörende nicht deine bereits existierende Meinung bestätigt.

    Krypto schrieb:

    Unser Universum hat offenbar eine dermaßen präzise Feinabstimmung, sodass sich die Vermutung aufdrängt, dass es unzählige andere, “versagte” Universen geben könnte.
    Wir können aber absolut keine fundierte Aussage darüber machen, ob und wenn ja, in was für einem Glücksfall wir hier leben.

    Du machst daraus:

    Damit drängt sich mir mehr denn je nach einem intelligenten Verursacher auf, der ein Meister im Fach ist.

    Also nochmal: “es drängt sich die Vermutung auf, dass es unzählige andere, “versagte” Universen geben könnte.
    Wenn sich dir daraus ein “Meister im Fach” “aufdrängt”, dann hast du noch keinen Meister gesehen. Dem sollte nämlich der überwiegende Teil seiner Stücke gelingen.

  174. #174 Lukas
    Wien
    29. November 2016

    Wenn er davon ausgehen würde das der meister experimentiert dann hat er nichts falsches gedacht.

    Wobei ich sagen muss dass das totaler Schwachsinn ist was er schrieb.

    Nur schade das es sich ausdehnt das macht ein intergalaktisches zusammenleben deutlich schwierigerXP

    Achja danke allen die versucht haben lehrend und ohne Formeln zu erklären. Macht Spaß sich oberflächlich zu informieren DANK EUCH^^

  175. #175 Lambda
    29. Juni 2017

    Mal eine Frage zu meinem Verständnis. Wenn ich annehme, dass das Universum wirklich unendlich ist und das wir innerhalb von 13,8 Milliarden Lichtjahren existieren, könnte es doch im Raum nach 13,8 Milliarden Lichtjahren Dinge geben, die wir nicht sehen können. Also z.B. schwarze Löcher. Vielleicht gab oder gibt es in diesem Raum alles nur kein Licht, aber große Anziehungen und damit Kollisionen die dann zu einer gigantischen Entstehung des uns bekannten sichtbaren Universums geführt hat. Jetzt wird die Gravitation nicht abgeschaltet und deswegen sieht es für uns so aus als würde sich alles, aus unserer Sicht, ausdehen. Im Grunde bewegt sich unser sichtbares Universum einfach nur in einem riesig großen Raum von Dingen die potenziel auch neue Universen entstehen lassen könnten. Ist es vielleicht so, dass es in dem nicht sichtbaren Bereich, das Unendliche, Anziehungen und Abstoßungen gibt, über unser Universum hinaus, ist wie eine große Masse aus Dingen, die sich verhält wie ein “lebender Organismus”?

  176. #176 Captain E.
    30. Juni 2017

    @Lambda:

    Mal eine Frage zu meinem Verständnis. Wenn ich annehme, dass das Universum wirklich unendlich ist und das wir innerhalb von 13,8 Milliarden Lichtjahren existieren, könnte es doch im Raum nach 13,8 Milliarden Lichtjahren Dinge geben, die wir nicht sehen können. Also z.B. schwarze Löcher. Vielleicht gab oder gibt es in diesem Raum alles nur kein Licht, aber große Anziehungen und damit Kollisionen die dann zu einer gigantischen Entstehung des uns bekannten sichtbaren Universums geführt hat. Jetzt wird die Gravitation nicht abgeschaltet und deswegen sieht es für uns so aus als würde sich alles, aus unserer Sicht, ausdehen. Im Grunde bewegt sich unser sichtbares Universum einfach nur in einem riesig großen Raum von Dingen die potenziel auch neue Universen entstehen lassen könnten. Ist es vielleicht so, dass es in dem nicht sichtbaren Bereich, das Unendliche, Anziehungen und Abstoßungen gibt, über unser Universum hinaus, ist wie eine große Masse aus Dingen, die sich verhält wie ein “lebender Organismus”?

    Nun, eher nicht. Das Universum ist auch fast sicher nicht unendlich groß. Die Mathematik kann ganz gut mit dem Begriff “unendlich” umgehen, aber die Physiker werden nervös, wenn die von ihnen verwendeten Formeln ein unendliches Resultat liefern. Normalerweise bedeutet das immer, dass sie irgendetwas noch nicht hinreichend gut genug verstanden haben. Allerdings schätzt man, dass das Universum tatsächlich so einen Durchmesser von 40-50 Milliarden Lichtjahren hat. Der Begriff “Durchmesser” bedeutet dabei aber nicht zwangsläufig, dass die Form des Universum eine Kugel sein muss. Vielleicht ist es auch ein Ellipsoid oder ein Torus.

    Letztlich würde es aber keinen Unterschied machen. Die Physik hat kosmologische Modelle aufgestellt, um das Universum zu beschreiben. Dabei kommt es vor allem darauf an, ob man mithilfe dieser Modelle die Vorgänge im Universum beschreiben, berechnen und vor allem voraussagen können. Das klappt insgesamt schon ganz gut. Sollten diese Modelle in Wirklichkeit grundfalsch sein, wäre das völlig egal, solange man keine neuen aufstellen kann, die bessere Ergebnisse liefern als die alten.

  177. #177 Spritkopf
    30. Juni 2017

    @Captain E.

    Allerdings schätzt man, dass das Universum tatsächlich so einen Durchmesser von 40-50 Milliarden Lichtjahren hat.

    Hast du dafür eine Quelle? Bei MartinB habe ich nämlich vor längerer Zeit einen Artikel gelesen, dass schon das (potentiell) sichtbare Universum einen Durchmesser von 93 Mrd. Lichtjahren hat. Und da es auf großen Skalen flach erscheint, wüsste ich nicht, wie man überhaupt eine begründete Abschätzung für den Durchmesser des Universums inklusive der für uns nicht sichtbaren Regionen geben könnte.

  178. #178 Alderamin
    30. Juni 2017

    @Captain E.

    Allerdings schätzt man, dass das Universum tatsächlich so einen Durchmesser von 40-50 Milliarden Lichtjahren hat.

    Das beobachtbare Universum hat einen Radius von 47 Milliarden Lichtjahren (mitbewegte Entfernung = Eigendistanz zum heutigen Zeitpunkt), muss aber erheblich größer sein, sonst würden wir es nicht als geometrisch flach messen (dass es an irgendeiner Kante kurz hinter’m Horizont aufhört, davon geht niemand aus). Bei Ethan Siegel steht, es habe mindestens einen Durchmesser, der 150-mal so groß ist wie der des beobachtbaren Universums, was etwa 14 Billionen Lichtjahren entspräche. Tatsächlich können wir nicht unterscheiden, ob es nur sehr groß ist oder unendlich, und die aktuelle Standardtheorie geht der Einfachheit halber von einem unendlichen Universum aus.

  179. #179 Alderamin
    30. Juni 2017

    @Spritkopf

    Du hast recht, Kommentar mit Quellen in der Mod.

  180. #180 Captain E.
    30. Juni 2017

    @Spritkopf:

    Hast du dafür eine Quelle? Bei MartinB habe ich nämlich vor längerer Zeit einen Artikel gelesen, dass schon das (potentiell) sichtbare Universum einen Durchmesser von 93 Mrd. Lichtjahren hat. Und da es auf großen Skalen flach erscheint, wüsste ich nicht, wie man überhaupt eine begründete Abschätzung für den Durchmesser des Universums inklusive der für uns nicht sichtbaren Regionen geben könnte.

    Die hatte ich noch so vage im Kopf, aber vielleicht ging es dabei ja auch um den Radius und nicht den Durchmesser?

    Außerdem expandiert das Universum bekanntlich – wahrscheinlich ist es einfach seither gewachsen. 😉

    Auf diesen Skalen wird aber alles komisch, wie etwa die Frage: Rotiert das Universum?

  181. #181 Herr Senf
    30. Juni 2017

    @Spritkopf #177
    “… da es auf großen Skalen flach erscheint, wüsste ich nicht, wie man überhaupt eine begründete Abschätzung für den Durchmesser des Universums … geben könnte.”
    Ich hab mal vor ein paar Jahren eine Abhandlung auf arxiv gelesen, an die ich mich dunkel erinnere, daß mit den bisherigen Meßgenauigkeiten die Befunde statistisch ausgewertet, das Universum mindestens 250 Hubble-Radien groß sein müßte, Endlichkeit möglich.
    Wir wären da bei einem Durchmesser von 7.000 Mrd Lj, 75x größer als der jetzige des sichtbaren Bereiches von 93 Mrd Lj. Aber das war nur eine Mindestabschätzung.

  182. #182 Alderamin
    30. Juni 2017

    Die Mindesta‌bschätzung ist, laut Eth‌an Sie‌gel (“Sta‌rts with a B‌ang”) 14 Billi‌onen LJ Durchm‌esser. Un‌endlich ist aber potenziell auch möglich und die einfa‌chste Ann‌ahme.

  183. #183 Spritkopf
    30. Juni 2017

    @Herr Senf

    Ich hab mal vor ein paar Jahren eine Abhandlung auf arxiv gelesen, an die ich mich dunkel erinnere, daß mit den bisherigen Meßgenauigkeiten die Befunde statistisch ausgewertet, das Universum mindestens 250 Hubble-Radien groß sein müßte

    Verstehe ich das richtig, dass dieses Papier dann ungefähr folgendes sagt?: “Bezogen auf die bisherigen Messungen darf eine Raumkrümmung (wenn sie denn vorliegt) höchstens den Wert X haben, damit sie in den Messungenauigkeiten verschwindet. Folglich muss ein Universum, wenn es endlich sein sollte, eine Mindestgröße von 7 Billionen Lichtjahren haben.”

    Eigentlich ist das lustig. Wir sind fast in der gleichen Situation wie ein Bewohner der mesopotamischen Tiefebene des Jahres 4000 v. u. Z., der auf das vor ihm liegende Land blickt, keine Erdkrümmung sehen kann und konstatiert, dass die Erde eine Scheibe sein muss. Mit dem Unterschied, dass er noch keine Fehlerrechnung beherrschte. 🙂

  184. #184 Captain E.
    30. Juni 2017

    @Alderamin:

    Das beobachtbare Universum hat einen Radius von 47 Milliarden Lichtjahren (mitbewegte Entfernung = Eigendistanz zum heutigen Zeitpunkt), muss aber erheblich größer sein, sonst würden wir es nicht als geometrisch flach messen (dass es an irgendeiner Kante kurz hinter’m Horizont aufhört, davon geht niemand aus). Bei Ethan Siegel steht, es habe mindestens einen Durchmesser, der 150-mal so groß ist wie der des beobachtbaren Universums, was etwa 14 Billionen Lichtjahren entspräche. Tatsächlich können wir nicht unterscheiden, ob es nur sehr groß ist oder unendlich, und die aktuelle Standardtheorie geht der Einfachheit halber von einem unendlichen Universum aus.

    Ah, richtig erinnert, aber falsch verstanden. Aber ob es nun 13,8 Milliarden Lichtjahre oder 14 Billionen Lichtjahre sind, was macht das schon aus? Ich meine, so ein Faktor 1000 unter Freunden… 😉

  185. #185 Frank Holek
    Rhinow
    14. April 2020

    ….. schneller je weiter es weg ist ? oder schneller je älter es ist ?
    kann dies nicht ein Denkfehler sein weil immer nur die “zunehmender Entfernung” nicht aber die “zunehmender Nähe zum Urknall” eine Rolle spielt.
    kann es sein dass Objekte, die älter und damit näher am “Urknall” sind schneller sein müssen – aber schneller als was ?

  186. #186 Frank Holek
    Rhinow
    14. April 2020

    die Rotverschiebung ist also nur der Beweis für die (zunehmende) Dehnung des Raumes – nicht für die zunehmende Geschwindigkeit von Objekten ?